Download general care of patients in therapy

Document related concepts

Medical ethics wikipedia , lookup

Adherence (medicine) wikipedia , lookup

Patient safety wikipedia , lookup

Electronic prescribing wikipedia , lookup

List of medical mnemonics wikipedia , lookup

Nurse–client relationship wikipedia , lookup

Transcript
General care of patients in therapy
Text test questions
1. A child experiencing a sudden shortness of breath, skin is cyanotic, curves extinction pulse at
the main vessels and myosis. What pershocher hovi measures?
A. Vnutrishnosertseve introduction of adrenaline
B. Oksyhenoterapiya
C. Intravenous eufilinu
D. Autolavage
E. * Ventilator closed cardiac massage
2. A child got in a car accident has signs of a damaged perfusion: sy stolichnyy pressure - 60 mm
Hg What is the best treatment?
A. Appointment of cardiac glycosides.
B. Dopamine infusion
C. Infusion of protein drugs
D. Appointment daturine
E. * Infusion salt solutions
3. A client in the face-down position is in which position?
A. Sims'
B. dorsal recumbent
C. lateral
D. * prone
E. No correct answer
4. A client in the face-down position is in which position?
A. Sims'
B. dorsal recumbent
C. lateral
D. * prone
E. No correct answer
5. A client is admitted to the emergency room for treatment of a severe laceration. The type of
assessment most likely to be performed by the nurse is
A. a comprehensive assessment
B. * an ongoing assessment
C. a focused assessment
D. an expansive assessment
E. No correct answer
6. A client is admitted to the emergency room for treatment of a severe laceration. The type of
assessment most likely to be performed by the nurse is
A. a comprehensive assessment
B. * an ongoing assessment
C. a focused assessment
D. an expansive assessment
E. No correct answer
7. A coal miner was affected when a gas mine blew up. In the dynamic observation dia
hnostovano effects of acute poisoning of mine gas syndrome toksyko-hypoxic encephalopathy.
What is the most appropriate treatment?
A. Cytochrome C
B. Forced diuresis
C. Exchange blood transfusion
D. * Hyperbaric oxygenation
E. Hemosorbtion
8. A comprehensive assessment is usually done
A. * upon admission to a health care facility
B. upon discharge from a health care facility
C. to focus in-depth on a particular health care problem
D. to provide systematic monitoring and observation related to specific problems
E. No correct answer
9. A comprehensive assessment is usually done
A. * upon admission to a health care facility
B. upon discharge from a health care facility
C. to focus in-depth on a particular health care problem
D. to provide systematic monitoring and observation related to specific problems
E. No correct answer
10. A decubitus ulcer is caused by
A. poor circulation
B. pressure on the skin
C. poor nutrition
D. none of the above
E. all of the above
11. A nurse is the leader of a cognitive behavior group. The group helps members learn about and
change problematic behaviors. The type of group the nurse is leading is a
A. self-help group
B. * task group
C. therapy group
D. therapeutic group
E. No correct answer
12. A nurse is the leader of a cognitive behavior group. The group helps members learn about and
change problematic behaviors. The type of group the nurse is leading is a
A. self-help group
B. * task group
C. therapy group
D. therapeutic group
E. No correct answer
13. A nurse must check a client's vital signs. Which personal space or invisible buffer zone will the
nurse be entering?
A. * intimate distance
B. close distance
C. personal distance
D. social or public distance
E. No correct answer
14. A nurse must check a client's vital signs. Which personal space or invisible buffer zone will the
nurse be entering?
A. * intimate distance
B. close distance
C. personal distance
D. social or public distance
E. No correct answer
15. A nurse taking part in therapeutic communication with a client promotes understanding of his
client's feelings and condition to enable the nurse and client to relate better. Which element of
therapeutic communication is the nurse practicing?
A. caring
B. empathy
C. * sympathy
D. validation
E. No correct answer
16. A nurse taking part in therapeutic communication with a client promotes understanding of his
client's feelings and condition to enable the nurse and client to relate better. Which element of
therapeutic communication is the nurse practicing?
A. caring
B. empathy
C. * sympathy
D. validation
E. No correct answer
17. A temperature of air (°C) and humidity (%) in the rooms of the hospital is:
A. 16-18°C, 45-50 %.
B. 17-19°C, 45-50 %.
C. 18-20°C, 50-55 %
D. 19-21°C, 50-55 %
E. 18-20°C, 50-55 %
18. Abnormally increased convexity of the curvature of the spine is documented as
A. lordosis
B. scoliosis
C. list
D. * kyphosis
E. No correct answer
19. Abnormally increased convexity of the curvature of the spine is documented as
A. lordosis
B. scoliosis
C. list
D. * kyphosis
E. No correct answer
20. Acceptance in a nurse-client relationship means
A. tolerating all the client's behaviors
B. * not expressing shock at a client's behaviors
C. adopting the client's value system
D. expecting the client to behave according to acceptable standards
E. No correct answer
21. Acceptance in a nurse-client relationship means
A. tolerating all the client's behaviors
B. * not expressing shock at a client's behaviors
C. adopting the client's value system
D. expecting the client to behave according to acceptable standards
E. No correct answer
22. Active listening
A. * focuses on the feelings of the individual who is speaking
B. occurs when the nurse is active in performing procedures for the client
C. involves responding in an active way to what the client is saying
D. means the nurse focuses primarily on the words of the speaker
E. No correct answer
23. Active listening
A. * focuses on the feelings of the individual who is speaking
B. occurs when the nurse is active in performing procedures for the client
C. involves responding in an active way to what the client is saying
D. means the nurse focuses primarily on the words of the speaker
E. No correct answer
24. After you intubate trachea clinic unexpectedly weak hypoxia. Breathing noises are only on the
right side. What complications due intubation?
A. Tracheal injury
B. * one lung intubation
C. Unexpected ekstubation
D. Left pneumothorax
E. Tube obstruction
25. All of the following are benefits of exercise EXCEPT
A. * decreased energy
B. improved bone density
C. lowered cholesterol
D. normalized glucose tolerance
E. No correct answer
26. All of the following are benefits of exercise EXCEPT
A. * decreased energy
B. improved bone density
C. lowered cholesterol
D. normalized glucose tolerance
E. No correct answer
27. All of the following are examples of focused questions EXCEPT
A. "Explain how the accident happened."
B. * "Are you in pain?"
C. "What is it about this headache that differs from others you have had?"
D. "What did your brother say to you before he passed out?"
E. No correct answer
28. All of the following are examples of focused questions EXCEPT
A. "Explain how the accident happened."
B. * "Are you in pain?"
C. "What is it about this headache that differs from others you have had?"
D. "What did your brother say to you before he passed out?"
E. No correct answer
29. All of the following data are collected from a client. Which is an example of objective data?
A. The client reports having a headache.
B. * The client's pulse is 88 beats per minute.
C. The client states that she is frightened.
D. The client states that she is having pain in her abdomen.
E. No correct answer
30. All of the following data are collected from a client. Which is an example of objective data?
A. The client reports having a headache.
B. * The client's pulse is 88 beats per minute.
C. The client states that she is frightened.
D. The client states that she is having pain in her abdomen.
E. No correct answer
31. All of the following statements about functional health patterns are true EXCEPT
A. * they are based on a particular theory about nursing
B. they focus on 11 functional health patterns
C. they allow gathering and clustering of information about the client's patterns
D. the coping-stress-tolerance pattern is one of the 11 patterns assessed
E. No correct answer
32. All of the following statements about functional health patterns are true EXCEPT
A. * they are based on a particular theory about nursing
B. they focus on 11 functional health patterns
C. they allow gathering and clustering of information about the client's patterns
D. the coping-stress-tolerance pattern is one of the 11 patterns assessed
E. No correct answer
33. Ambulance deliver Lena patient 52 years. Unconscious, the skin and mucous membranes dry.
Breath superficial, often CHSS-136/min., AT-70/30 mm.rt.st., TSVT-0 mm vod.st., N-160 g / l,
No-0, 64, glucose - 47, 2 mmol / l, sodium - 168 mmol / L, potassium - 6.3 mmol / l,
osmolarity, 382 mOsm / liter. What is the third infusion solutions ba pour first?
A. Solution Ringera
B. 0.9% NaCl solution
C. Polihlyukin
D. * 0.45% NaCl solution
E. 15% solution of glucose
34. Ambulance ka through several minutes after the call arrived at the scene of accidents. On the
sidelines child 14 years of multiple, independent not breathing incorporated stop cardiac
activity. Define the right balance between the number of contractions and chest number
vdyhuvan air during cardio-pulmonary resuscitation:
A. 6:1
B. 7:1
C. 3:1
D. * 5:1
E. 4:1
35. An example of a block to creative thinking is
A. * fear of making a mistake
B. open mindedness
C. self-confidence
D. self-evaluation
E. No correct answer
36. An example of a block to creative thinking is
A. * fear of making a mistake
B. open mindedness
C. self-confidence
D. self-evaluation
E. No correct answer
37. An example of communication through the visual channel is
A. counting a client's pulse
B. noting the client's verbal response to a question
C. giving the client a back massage
D. * observing that the client looks away from the nurse when discussing a certain subject
E. No correct answer
38. An example of communication through the visual channel is
A. counting a client's pulse
B. noting the client's verbal response to a question
C. giving the client a back massage
D. * observing that the client looks away from the nurse when discussing a certain subject
E. No correct answer
39. An important way to reduce the incidence of decubitus ulcers is to:
A. keep the client in bed
B. force fluids every 2 hours
C. all of the above
D. none of the above
E. change position every 2 hours
40. An important way to reduce the incidence of decubitus ulcers is to:
A. keep the client in bed
B. force fluids every 2 hours
C. change position every 2 hours
D. none of the above
E. all of the above
41. An unconscious client who has just been involved in a motor vehicle accident is brought to the
emergency department. Which presenting clinical manifestation makes the nurse suspicious of
an opioid overdose rather than increased intracranial pressure as a cause of the
unconsciousness?
A. * Pinpoint pupils
B. Respiratory depression
C. Hyporeflexive deep tendon reflexes
D. Evidence that the client has vomited
E. No correct answer
42. As a nurse, you are caring for Mr. J. His is an immigrant from BosniA. Mr. J. speaks no
English. You obtain the services of an interpreter. What is the most effective way for you to
work with an interpreter?
A. Avoid use of pictures or gestures to clarify meaning of words.
B. Speak to the client rather than to the interpreter.
C. * Use various interpreters for interactions with Mr. J. to allow him to become familiar
with many people.
D. Direct your conversation to the interpreter who will then translate the information to
Mr. J.
E. No correct answer
43. As a nurse, you are caring for Mr. J. His is an immigrant from BosniA. Mr. J. speaks no
English. You obtain the services of an interpreter. What is the most effective way for you to
work with an interpreter?
A. Avoid use of pictures or gestures to clarify meaning of words.
B. Speak to the client rather than to the interpreter.
C. * Use various interpreters for interactions with Mr. J. to allow him to become familiar
with many people.
D. Direct your conversation to the interpreter who will then translate the information to
Mr. J.
E. No correct answer
44. Bed linen for patients must be changed:
A. Not longer than once every 15-16 days.
B. Not longer than once every 10 days.
C. Not longer than once every 12 days.
D. Not longer than once every 14 days.
E. Not longer than once every 7 days.
45. Bedsores may develop in clients who are
A. incontinent
B. poorly nourished
C. all of the above
D. none of the above
E. paralyzed
46. Before assisting a client into a wheelchair, the FIRST action would be to check if the
A. client is adequately covered
B. floor is slippery
C. door to the room is closed
D. wheels of the chair are locked
E. none of the above
47. Before assisting a client into a wheelchair, the FIRST action would be to check if the
A. client is adequately covered
B. floor is slippery
C. door to the room is closed
D. none of the above
E. wheels of the chair are locked
48. Before taking a temperature with a mercury-in-glass thermometer, you should shake the
thermometer to move the mercury:
A. at the level of normal body temperature
B. at the level of 37 ?C
C. at the level of 40 ?C
D. at the level of 0 ?C
E. below normal body temperature
49. Before taking a temperature with a mercury-in-glass thermometer, you should shake the
thermometer to move the mercury:
A. * below normal body temperature
B. at the level of normal body temperature
C. at the level of 37 °C
D. at the level of 40 °C
E. at the level of 0 °C
50. Boy 7 years when bathing in the lake and drowned. After 2 minutes he was pulled out of the
water. OBJECTIVE: cyanosis of skin, swelling neck veins, the allocation of the mouth and
nose frothy liquid, not breathing. What needs to be implemented in the first place?
A. * Resume permeability of respiratory tract
B. Perform CPR
C. Perform indirect heart massage
D. administer Lasix intravenously 1 mg / kg body weight
E. Intravenous fluid to enter 5 ml 2.4% solution eophilin
51. Brigade ambulance that arrived at the scene of accident 2 mins after it happend , noted in 5year old child without lung function and blood circulation, so was launched the initial
reanimation moves in accordance with accepted around world the "ABC rule”. What is it in
essence?
A. The need to strictly adhere to a specific sequence of reanimation measures by analogy
with the letters of the alphabet
B. In a certain primary reanimstion events
C. * In the integrated application of three methods, the first letters of names which
shortens now look like "ABC"
D. Rule ABC is not related to resuscitation.
E. In need of a population, as the alphabet, rules of primary care
52. Child 10 years brought on to shore in 3 minutes after drowning. OBJECTIVE: pulse on
peripheral artery not determined pupil moderately developed weakly responsive to light. Skin
pale with a cyanotic tint. In the mouth sand, silt, breathing sharply suppressed. What are the
urgent and immediate action in providing assistance to the affected?
A. * Restoration cross the upper respiratory tract
B. Closed cardiac massage
C. Calling specialized health care team
D. Attach affected on the horizontal surface of well
E. Artificial pulmonary ventilation by "mouth to mouth"
53. Child 4 months conducted artificial lung ventilation mode normoventelation on the acute
respiratory insuficiency that caused acute bronchiolitis. One of the elements of control
performance monitoring CPR is a free cross endotracheal tube. Which of these symptoms
showed its obstruction?
A. Cyanosis
B. Cough
C. Tachycardia
D. Reducing pressure on inspiratoration
E. * Increasing pressure on inspiratoration
54. Child in olihoanurychniy stage of acute renal failure, on the background hlomeru lonefrytu,
unconscious. OBJECTIVE: midriaz, no peripheral pulse, surface bradypnoe. Potassium serum 9.0 mmol / l, the ECG signs of ventricular fibrillation. What is urgent for medical walk?
A. Peritoneal dialysis
B. Furosemide, adrenaline intravenously
C. * Defibrillation, the introduction of soda, calcium chloride
D. Precardial strike
E. Lidocaine intravenous
55. Clients on which type of medications are at highest risk for development of necrosis of the
femur head?
A. phenothiazines
B. corticosteroids
C. * amphetamines
D. anticoagulants
E. No correct answer
56. Clients on which type of medications are at highest risk for development of necrosis of the
femur head?
A. phenothiazines
B. corticosteroids
C. * amphetamines
D. anticoagulants
E. No correct answer
57. Clinical death in conditions normothermia last:
A. 7-12 minutes
B. 1-3 minutes
C. * 4-5 minutes
D. 5-7 minutes
E. 10 minutes
58. Communication experts estimate that the majority of communication occurs through
A. * nonverbal messages
B. paralinguistic cues
C. paraverbal cues
D. verbal messages
E. No correct answer
59. Communication experts estimate that the majority of communication occurs through
A. * nonverbal messages
B. paralinguistic cues
C. paraverbal cues
D. verbal messages
E. No correct answer
60. Conversion of Centigrade to Fahrenheit is
A. Degrees F = 9/5 (?C) + 32
B. Degrees F = 7/5 (?C) + 32
C. Degrees F = 7/5 (?C) + 52
D. Degrees F = 9/5 (?C) + 45
E. Degrees F = 9/5 (?C) + 40
61. Conversion of Fahrenheit to Centigrade is
A. Degrees C = 8/9 (?C - 40)
B. Degrees C = 5/9 (?C - 32)
C. Degrees C = 3/5 (?C - 43)
D. Degrees C = 9/5 (?C - 52)
E. Degrees C = 8/5 (?C - 32)
62. During CPR you must check the presence of breathing and pulse every
A. 1 min
B. 3 min
C. 10 min
D. 7 min
E. 5 min
63. During the nasoenteral tube insertion you see some changes in patient's respiratory status. What
is you action?
A. withdraw the tube
B. stop insertion for some time
C. give some water to the patient
D. call the doctor
E. continue insertion
64. During the resuscitation solution sodium hydrocarbonate imposed in order to:
A. * correction of metabolic acidosis
B. Prevention of respiratory acidosis
C. eliminate hypoxia
D. treatment of metabolic alkalosis
E. increase of survival of the brain under conditions of hypoxia
65. E 6
A. 2
B. 3
C. 5
D. 4
66. Equipment used for prevention of the pressure ulcers
A. pillows, rolled-up blankets or towels
B. egg-crate mattress
C. specialty beds, heel and elbow protectors
D. lotion or powder, soap and water
E. all listed above
67. Female 12 years was pulled out of water in 3 minutes after drowning in the river. Parents
proformed CPR on her until the ambulance came. They managed to achieve some rare heart
beats, but not breathing resumed. What should the doctor do, immediately:
A. "Triple reception” for P. Safaris;
B. * Intubate trachea, clear the resparitory tract, and do CPR;
C. Indirect heart massage;
D. Intercardiac injection of adrenalin;
E. Intercardiac injection atropine sulfate.
68. Female 18 years old, while diving into the water, hit head against the bottom of the river. Now
has weakness in upper limbs, pain in the cervical spine. Involuntary position - head aligned
right. Sharp pain in cervical area. Sensitivity is not changed. What medical personal must
provide more assistance?
A. * Enter a pain medication, lay collar , transported to special Fixtures "Specialized office
hard stretcher
B. Put the patient on a soft litter, and the head and neck pillow and put the trans to hospital
C. Enter the heart and respiratory analeptic and hold the rigid litter with a pipe under the
veins
D. Enter the pain and transported to hospital in
69. Female 32 years enjoyed not good appliances. Suddenly fell, unconscious, had convulsions.
Affecting heart rate, most likely was dut marked electrocardiographic?
A. Asystole
B. Paroxysmal tachycardia
C. Atrioventricular block
D. * Ventricular fibrillation
E. Mechanical ventricular
70. Female 64 years old with arrhythmia when suddenly fainted. Another doctor at the review
konstatova to faint, the lack of Ripple on a.sarotis and heart tones, narrow pupil and family
superficial respiration. What, in this case must begin reanimation measures?
A. * As is the introduction of atropine
B. Fisticuff on sternum
C. Conducting cardiostimulation
D. Intubation and holding CPR
E. As is the introduction of adrenaline
71. Fever is generally a temperature above
A. 38,2 C°
B. 38,4 C°
C. 38,6 C°
D. 38 C°
E. 39 C°
72. For an intracardiac injection needle is injected at:
A. 5 th intercostal space, at the top of the heart projection
B. * 3 rd intercostal space, left from sternum
C. on the lower edge of the 4 th rib, left of sternum
D. 4 th intercostal space at 1.5 cm left of sternum
E. in place of projection beat
73. For the intensive infusion therapy in patients with acute respiratory insufficiency of a
catheterization of subclavian veins for Seldinherom. After introducing 600 ml of infusion fluid
state of the patient worsened dramatically increased tachiapnoe from 26 to 40 in 1 minute, with
auscultation breath right sharply weakened, on percution - dullness. Your diagnosis?
A. Wet lung
B. Brain edema
C. * Hydrothrax
D. Pulmonary artery thromboembolism
E. Acute heart failure.
74. For the number of measures for providing a protective mode we should perform the following
A. Control of adopting medicines
B. Regular ventilation of wards.
C. Wet cleaning of the departments
D. Removal of external irritants and negative emotions.
E. Prophylaxis of bedsores
75. How are stress and substance abuse interrelated?
A. * The neurotransmitters stimulated by many abused substances enhance the sense of
well-being and cause the sensation of stress when the substance wears off.
B. Chronic substance abuse leads to destruction of brain cells in the limbic area, causing
an increased intensity of the stress response.
C. Most abused substances are perceived by the body as physiologic stressors, which result
in a heightened “fight-or-flight” response.
D. As tolerance or addiction develop, more of the abused substance is needed to achieve
the same pleasurable responses.
E. No correct answer
76. How can you determine the required length of a nasogastric tube?
A. by measuring from bridge of nose to umbilicus
B. by measuring from bridge of nose to earlobe and to xiphoid process of sternum
C. by measuring from mouth to umbilicus
D. by measuring from earlobe to pubis
E. by measuring from bridge of nose to xiphoid process of sternum
77. How many minutes can the brain survive without oxygen in normal conditions?
A. 3-5 min
B. 1-3 min
C. 5-8 min
D. 7-10 min
E. 4-6 min
78. How many rescue breaths per minute do you have to give for the victim if he has a pulse but is
not breathing spontaneously?
A. 7
B. 15
C. 20
D. 10
E. 5
79. how much should we increase the voltage each discharge during defibrilation?
A. 1000 V
B. * 500 in
C. 250 in
D. 20 in 1 kg of body weight
E. stress can not boost
80. How often should the ward of the therapeutic department be cleaned up?
A. as often as necessary
B. every morning
C. every evening
D. it doesn’t need any cleaning
E. twice per day
81. If an assessment of a client takes place "in the field", the nurse must be aware of all of the facts
EXCEPT
A. * the nurse would not be liable if they did not begin to give care
B. the Good Samaritan Law applies
C. the nurse must stay with the client until another qualified health care provider arrives
D. the Abandonment Law applies
E. No correct answer
82. If an assessment of a client takes place "in the field", the nurse must be aware of all of the facts
EXCEPT
A. * the nurse would not be liable if they did not begin to give care
B. the Good Samaritan Law applies
C. the nurse must stay with the client until another qualified health care provider arrives
D. the Abandonment Law applies
E. No correct answer
83. If body temperature is increased 1 ?C, the pulse rate will be:
A. The same as in normal temperature
B. Decreased 8-10 beats
C. Increased 20-25 beats
D. Decreased 20 beats
E. Increased 8-10 beats
84. If body temperature is increased 1 ?C, the pulse rate will be:
A. The same as in normal temperature
B. * Increased 8-10 beats
C. Decreased 8-10 beats
D. Increased 20-25 beats
E. Decreased 20 beats
85. In a closed heart massage adults brush bottom hand should be placed on:
A. on sternum, fingers toward the chin
B. * on the bottom third of sternum, fingers parallel ribs
C. on the middle third of sternum
D. 4 the intercostals space, left from sternum
E. breastfeeding cage - in place of projection top heart
86. In a drowning in sea water in patients has clinical death. Specify the correct sequence
reanimation these procedures:
A. * Restoration of function of upper respiratory tract, CPR indirect cardiac massage;
B. Indirect heart massage, CPR, restoring function upper respiratory ways;
C. CPR, indirect heart massage, rehabilitation upper cross-halnyh ways;
D. Indirect heart massage, CPR;
E. Electric defibrilation, heart CPR.
87. In a respirator «mouth to mouth» need to inject air into volumes?
A. 1700-2000 ml
B. 500-700 ml
C. 1200-1600 ml
D. * 800-1100 ml
E. make maximum inspiratory
88. In assessing the patient's body temperature, it was found to be 38,5 ?C. This is:
A. Normal
B. Decreased
C. Subfebril
D. Elevated
E. All are correct
89. In caring for a confused elderly man, it is important to:
A. close the door to the room so that he does not disturb other patients
B. keep the room dark and quiet at all times to keep the patient from becoming upset
C. remind him each morning to shower
D. keep the bedrails up except when you are at the bedside
E. all of the above
90. In caring for a confused elderly man, it is important to:
A. close the door to the room so that he does not disturb other patients
B. keep the room dark and quiet at all times to keep the patient from becoming upset
C. remind him each morning to shower
D. keep the bedrails up except when you are at the bedside
E. all of the above
91. In patient A., 47 years diagnosed with clinical death, the ECG - asystolia. In the background
pharmacological measures should be first given:
A. Atropine
B. Lidocain
C. Calcium chloride
D. Sodium hydrogen
E. * Adrenalin
92. In patients 30 years after a road accident complaint appeared to harsh wind. OBJECTIVE: pale
skin, cyonotic. Subcutaneously emphyzema in the area of the chest, abdomen, neck on the right
side. On auscultation breathing thing does not happen on the right side ; heartrate - 130/min.,
AP - 80/60 mm RT. art., TSVT -140 mm water. art., CHD - 30/min., Ht - 0,27, Hb - 90 g / liter.
Further treatment in the first place should include the following measures:
A. Urgent CPR
B. Massive infusion therapy KRI staloyidnymy solutions
C. Dopamine infusion, 2-5 mg / khhhv
D. * Empty pleural puncture on the right
E. Oxygenation 100% oxygen
93. In patients 65 years with acute myocardium etc. farktom on 2nd day of ill-tion developed
complete atrioventricular block with CHSS-32/min. What drug is shown to provide emergency
assistance?
A. Eufilin
B. * Atropine
C. Prednisolone
D. Miofedrin
E. Nifedypin
94. In patients 85 years of acute pain during electrocardioanalysis observed signs of clinical death
those recorded asystole with preserving transport function. Medical measures for 3 minutes
ineffective. Which of these measures will be most effective him in this situation? .
A. Correction of metabolic acidosis
B. Defibrillisation
C. Adrene stimulation
D. * Electrocardiostimulation
E. Precardial strike
95. In patients with symptoms of lack of circulation, have asystole on ECG. The primary
therapeutic measure should be:
A. Introduction lidocaine
B. * Introduction adrenaline
C. Introduction daturine
D. Introduction of calcium chloride
E. Electric defibrillation
96. In the communication process, encoding is defined as
A. a stimulus produced by a sender and responded to by a receiver
B. the person who intercepts the sender's message
C. a person who generates a message
D. * the use of language or other signs and symbols for sending messages
E. No correct answer
97. In the communication process, encoding is defined as
A. a stimulus produced by a sender and responded to by a receiver
B. the person who intercepts the sender's message
C. a person who generates a message
D. * the use of language or other signs and symbols for sending messages
E. No correct answer
98. In the patient suddenly came sharp pain behind the sternum with irradiation in the left hand.
OBJECTIVE: agitated, pale skin covers, CHDR-38/min, AT-180/110 mm Hg During the
review loses consciesness, and fall. Pulse on the main vessels is not determined pupil uniformly
expanded. What priority actions?
A. Add to / in central analeptic
B. Call ambulance station
C. Do nothing, because death is the
D. * Start a complex cardio-pulmonary and cerebral resuscitation
E. administer mezaton
99. In what organ is the thermoregulatory center located?
A. liver
B. brain
C. fat tissue
D. intestine
E. heart
100.
In what organ is the thermoregulatory center located?
A. liver
B. heart
C. * brain
D. fat tissue
E. intestine
101.
In women in 18th week of pregnancy you stop feeble circulation due herbs us. In what
position should be held indirect heart massage?
A. * With raised feet pregnant
B. In napivsydyachomu position
C. With sunken feet pregnant
D. In pregnant position on the left side
E. In pregnant position on the right side
102.
Increase of body temperature is called:
A. Hypothermia
B. Hyperthermia
C. Arrhythmia
D. Poliuria
E. Hypertension
103.
Increase of body temperature is called:
A. * Hyperthermia
B. Hypothermia
C. Hypertension
D. Arrhythmia
E. Poliuria
104.
Male 10 years old received by the division of resuscitation and intensive therapy in
comatose condition. Breath spontaneous, superficial, involving supporting muscles part take in
the act of breathing. CHDR-32/min. perioral cyanosis. What is noticeable is in the artificial
tylyatsiyi pulmonary veins in patients in the unconscious state?
A. Calculation of ability to be affected
B. Respiration rate
C. Feeling sick state comfort
D. Spirometrychni measuring respiratory volume
E. * Arterial blood gas analysis
105.
Male 30 years as a result of a accident. Consciousness absent. Pulse on carotid not
palpable, no breathing. At waist level in the affected skin a re min. What are the steps you need
to start?
A. Immediate start holding of pulmonary ventilation and external cardiac massage without
losing time
B. Turn right on the affected side
C. Do not touch affected to at existence of GA
D. Conclude the affected back on board
E. * Conduct artificial lung ventilation and external heart massage the previous removal
belt
106.
Male 30 years old at the time when he was eating suddenly grabbed himself by the
neck, was cyanosis, inability to make a breath, lack of voice, coughing. What should be done
first?
A. Tracheotomy;
B. Conicotomiyu;
C. palpate the inside of the mouth;
D. Intubation of trachea;
E. * Heimlich maneuver.
107.
Male 46 years delivered to the department of cardiology care pain angious character,
who appeared for the first time in life 2 hours ago. In the office there was a memory pink
circulation and respiration. Started cardiopulmonary resuscitation. In monitoring the frequency
reductions 212/hz ventricular complex SIK8 deformed and rozshyrenyy, ST segment
depression. What you want to start healing the patient?
A. Pacemaker implantation
B. Intravenous adrenaline
C. Intravenous novocayinamide
D. * Countershock
E. Intravenous lidocaine
108.
Male 49 years old complains of chest pain, which appeared first as dynu during fast
passages. Objective no: moderate tachycardia, AT-150/90 mm Hg While recording ECG
suddenly unconscious, a single tonic convulsion CKE's summer muscles, liquid breathing,
irregular, pulse on the carotid arteries is absent. What urgent action to provide assistance hi?
A. Off electrocardiograph
B. Define the level of blood pressure
C. Enter protysudomni intravenous drugs
D. * Run precardial strike
E. Perform defibrillisation
109.
Male 50 years old was injured by the accident. Unconscious. Since the injury took 3
minutes. What is the primary symptom for the diagnosis of stopping blood circulation?
A. Lack of spontaneous rhythmic breathing
B. Lack of heart tones during AU skultatsiyi
C. Advanced pupil, no reaction to light
D. * Lack of pulse in a.sarotis
E. Lack of pulse in the peripheral arteries
110.
Men 57 years on the trolley bus stop suddenly became ill, lost consciesness, the skin
became cyonotic. Not Breathing pulse not determined. Started holding indirect heart massage
and artificial ventilation of lungs. How should be placed hands of the reanimator?
A. In 2 cross finger right of sternum 4 th intercostal space
B. In 2 cross finger left sternum 4 th intercostal space
C. In the middle third of sternum
D. behind the back
E. * In 2 cross finger machine-like above sternal articulation
111.
Microorganisms can be spread by direct and indirect contact. An example of indirect
contact is
A. bathing the patient
B. using contaminated blood
C. breathing dust particles in the air
D. touching objects or dirty instruments
E. none of them
112.
Mild anxiety affects the communication process by
A. severely restricting the perceptual field
B. blocking perception
C. * broadening the perceptual field
D. distorting the perceptual field
E. No correct answer
113.
Mild anxiety affects the communication process by
A. severely restricting the perceptual field
B. blocking perception
C. * broadening the perceptual field
D. distorting the perceptual field
E. No correct answer
114.
Moving the extremity laterally and away from the midline of the body is
A. flexion
B. adduction
C. extension
D. * abduction
E. No correct answer
115.
Moving the extremity laterally and away from the midline of the body is
A. flexion
B. adduction
C. extension
D. * abduction
E. No correct answer
116.
Nasal temperature measuring is:
A. safe and accurate
B. unsafe and accurate
C. safe and not accurate
D. * not applicable
E. applicable only in patients without teeth
117.
Nasal temperature measuring is:
A. safe and accurate
B. unsafe and accurate
C. safe and not accurate
D. applicable only in patients without teeth
E. not applicable
118.
Older man in the street suddenly became unconscious, continuing to breathe
independently. What is the event?
A. Hold heart massage
B. Extract from the mouth of dentures and other foreign body
C. Put on the floor, raise the lower limbs
D. * Slush precardial strike
E. Hold Ventilator
119.
On the pavement lay unconscious man 30-35-years. OBJECTIVE: pulse on peripheral
arteries are not determined on carotid pulse frequent, weak content, independent of breath 20/min. What will be true?
A. * Provide patient position on the side face down and call an ambulance
B. To bring the patient closer to the super clinic
C. Perform CPR and call an ambulance
D. Perform only indirect heart massage
E. Not hayuchy time cause rapid assistance can
120.
Oral route temperature measuring should be delayed if:
A. patient has smoked
B. patient has ingested hot meals
C. patient has ingested cold meals
D. all mentioned reasons
E. no reasons for delay
121.
Oral route temperature measuring should be delayed if:
A. patient has smoked
B. patient has ingested hot meals
C. patient has ingested cold meals
D. * no reasons for delay
E. all mentioned reasons
122.
Oral temperature is:
A. lower than axillary
B. the same as axillary
C. the same as rectal
D. higher than axillary
E. all are correct
123.
patient 12 years of age you pull out of cold sea water after 15 minutes after drowning
without signs of life. What steps need to begin?
A. * Exempt airways of water, a drainage provisions, to initiate moves for cardiopulmonary resuscitation (CPR);
B. Do not do SLR, due to the large time interval after the drowning;
C. Vehicle suffered local medical institution for the SLR;
D. Do not waste time on the removal of water to conduct CPR;
E. conduct conicotomia.
124.
Patient 35 years in the intensive care forth day after hemorrhage and hemorrhagic
shock. Daily diuresis - 50 ml. Sick euphoric, not adequately evaluate its condition. The second
day marked vomiting, for ECG beat; TSVT - 159 mm water. art., CHD - 32/min. What
complications developed in patient?
A. Acute heart failure
B. Posthypoxic encephalopathy
C. Acute respiratory failure
D. * Acute renal failure of prerenal genesis
E. Acute delay of diureses
125.
Patient 57 years delivered to the receiving Office kardialhiyamy and widespread ST
segment depression on ECG. Suddenly unconscious, the pulse in the central arteries is not
defined with nytsi narrow. Immediately started cardio-pulmonary resuscitation. What it
measures begins?
A. Setting an artificial driving rhythm
B. Sublingual nitroglycerin
C. * Heart massage, artificial respiration, mechanical and electrical defibrillation
D. Intravenous lidocaine
E. Intacardial introduction of adrenaline
126.
Patient aged 78 years, during the review in the admissions office, suddenly znepry
languishing. Breathing and pulse of the carotid arteries is not defined. When trying for
bezpechyty permeability airways for artificial respiration, turn vyy doctor found that the
patient's head zakydannya almost impossible due to malformed cervical spine. What additional
steps required for these ob stavyn?
A. * Raise the lower jaw and open the patient up his mouth, with moderate head deviation
B. Rotate patient's head toward him and open mouth
C. Put a pillow under the head of the patient
D. Make additional efforts to maxi malnoho zakydannya head sick
E. Put a pillow or folded towel under the patient shoulders
127.
Patient in the house suddenly became unconscious, no breathing, pulse on the carotid
arteries is not defined. What should you start doing?
A.
B.
C.
D.
E.
Conducting external cardiac massage
Introduction adrenaline
Precardial strike
Introduction daturine
* Restoration of respiratory shlya cross hiv
128.
Patients in a state of clinical death. conducted on artificial lung ventilation noise dom
from mouth to mouth and indirect heart massage. The doctor pointed out that the prostitute trya
not pass in the airways of patients th, and his head and torso are in the same plane. What is the
cause of inefficient sti respirator in that situation?
A. Small mouth of the patient
B. Holding indirect heart massage
C. A small amount of air that inspired
D. Lack of probe in the stomach
E. * Swallowing of tongue
129.
Pediculosis evaluation should be done in:
A. surgical department
B. outpatient department
C. ambulance
D. * Admission department
E. at home
130.
Rectal temperature is:
A. * higher than axillary
B. lower than axillary
C. the same as axillary
D. the same as oral
E. lower than oral
131.
Rectal temperature is:
A. lower than axillary
B. higher than axillary
C. the same as oral
D. lower than oral
E. the same as axillary
132.
Reliable sign of stopping cardiac activity:
A. * isolines on ECG
B. loss of conscisness
C. pronounced cyanosis of skin and mucous
D. pulse over the artery ray stopped being able to palpated
E. lack of blood pressure
133.
Rotating the forearm laterally at the elbows so the palm of the hand turns laterally to
face upward is called
A. * supination
B. internal rotation
C. pronation
D. external rotation
E. No correct answer
134.
Rotating the forearm laterally at the elbows so the palm of the hand turns laterally to
face upward is called
A. * supination
B. internal rotation
C. pronation
D. external rotation
E. No correct answer
135.
Sterilization means
A. * Destroying all living organism and bacterial spores
B. Destroying all living organism without bacterial spores
C. Destroying Gramm-positive organism
D. Destroying Gramm-negative organism
E. Any from all previous propositions
136.
suddenly unconscious, there was a one time convulsion tonic skeletal muscles breathing
is irregular, pulse on the carotid arteries is absent. To provide assistance to first:
A. Perform defibrillation
B. administer anticonvulsant intravenous drugs
C. Off electrocardiograph
D. Define the level of blood pressure
E. * Perform precardial push and begin cardiopulmonary resuscitation.
137.
The basis of a person's philosophy of life is her
A. culture
B. value system
C. * ethnicity
D. educational level
E. No correct answer
138.
The basis of a person's philosophy of life is her
A. culture
B. value system
C. * ethnicity
D. educational level
E. No correct answer
139.
The boy 2 years after the consumption of marine food fish, the night suddenly appeared
and pronounced swelling of the face and tongue, inspiratory dyspnoe, diffuse cyanosis, which
periodically changes apnea. What are initial emergency measures?
A. Introduction of glucocorticosteroids
B. Autolavage
C. Oxygen through nasal catheters
D. Dehydration therapy
E. * conicotomia
140.
The child stops the heart. After intubation and subsequent CPR it porozhevishala, heart
rate recovered to 40/min, systolic SC-50 mm Hg What is the optimal future treatment?
A. Infusion salt solutions
B. Appointment of cardiac glycosides
C. Dopamine infusion
D. * Appointment daturine
E. Synchronized kardioversiya
141.
The client has been transferred to the medical-surgical unit from the emergency
department. His admitting diagnosis is “barbiturate overdose.” What is the nurse’s priority
intervention?
A. Performing neurologic checks every 4 hours
B. Providing emotional support
C. Restricting visitors to immediate family
D. * Taking vital signs every 4 hours
E. No correct answer
142.
The client has narcolepsy. The nurse should be prepared to teach the client about which
drug category that has potential for abuse?
A. * Amphetamines
B. Benzodiazepines
C. Barbiturates
D. Phencyclidines
E. No correct answer
143.
The client smokes three packs of cigarettes per day. The nurse should teach this client
about his or her increased risk for which chronic health problem as a result of tobacco abuse?
A. Chronic pancreatitis
B. Rheumatoid arthritis
C. * Cardiovascular disease
D. Type 2 diabetes mellitus
E. No correct answer
144.
The fifth hierarchy of needs according to Maslow is
A. self-esteem
B. need for love and belonging
C. safety and security
D. * self-actualization
E. No correct answer
145.
The fifth hierarchy of needs according to Maslow is
A. self-esteem
B. need for love and belonging
C. safety and security
D. * self-actualization
E. No correct answer
146.
The first step in becoming nonjudgmental is to
A. learn more about other people
B. accept your own feelings
C. identify the source of your negative feelings
D. * recognize that one's thoughts are biased and prejudicial
E. No correct answer
147.
The first step in becoming nonjudgmental is to
A. learn more about other people
B. accept your own feelings
C. identify the source of your negative feelings
D. * recognize that one's thoughts are biased and prejudicial
E. No correct answer
148.
The following are data from a client. Which is an example of subjective data?
A. * The client reports having nausea.
B. The client's blood pressure is 120/82.
C. The client has a reddened area on the buttocks.
D. The client's blood sugar is 100 g/dl.
E. No correct answer
149.
The following are data from a client. Which is an example of subjective data?
A. * The client reports having nausea.
B. The client's blood pressure is 120/82.
C. The client has a reddened area on the buttocks.
D. The client's blood sugar is 100 g/dl.
E. No correct answer
150.
The function of feedback in communication is to
A. send a clear message
B. * provide the sender with information about the receiver's perception of the information
C. help the receiver decode the message
D. assist the sender in encoding the message
E. No correct answer
151.
A.
B.
C.
D.
E.
152.
A.
B.
C.
D.
E.
153.
A.
B.
C.
D.
E.
154.
A.
B.
C.
D.
E.
155.
A.
B.
C.
D.
E.
156.
A.
B.
C.
D.
E.
157.
A.
B.
C.
D.
E.
158.
A.
B.
C.
D.
E.
159.
A.
B.
C.
The function of feedback in communication is to
send a clear message
* provide the sender with information about the receiver's perception of the information
help the receiver decode the message
assist the sender in encoding the message
No correct answer
The highest body temperature is temperature measured in the:
axilla
oral cavity
nose
rectum
all are equal
The highest body temperature is temperature measured in the:
axilla
* rectum
oral cavity
nose
all are equal
The most common location for assessing the pulse is:
* Radial artery
Brachial artery
Femoral artery
Over the heart
Temporal artery
The MOST serious problem that wrinkles in the bedclothes can cause is
restlessness
sleeplessness
decubitus ulcers
shock
bleeding
The MOST serious problem that wrinkles in the bedclothes can cause is
restlessness
sleeplessness
bleeding
shock
decubitus ulcers
The normal temperature of the human body is :
36,0-36,5 C°
36,0-36,6 C°
36,2-36,6 C°
36,2-37,1 C°
36,2-37,0 C°
The nurse fits crutches correctly when
the crutch pad is one inch below the axilla
* the handgrip is adjusted to allow the client to have elbows bent at 30 degrees
the crutches force the client to stand upright
the handgrip is adjusted to allow the client to have elbows nearly straight
No correct answer
The nurse fits crutches correctly when
the crutch pad is one inch below the axilla
* the handgrip is adjusted to allow the client to have elbows bent at 30 degrees
the crutches force the client to stand upright
D. the handgrip is adjusted to allow the client to have elbows nearly straight
E. No correct answer
160.
The nurse in an admitting department is going to measure the temperature of the patient
with fever. She cleaned the inguinal area with alcohol before manipulation. Explain why this
procedure has been done:
A. For the purpose of providing a more exact measuring of temperature.
B. For the best fixing of thermometer in ingiunal area.
C. For the purpose of providing a hygienical regime of procedure.
D. There is no right answer
E. All answers are correct
161.
The nurse is assessing a client who has been immobilized for several weeks. What may
occur as a result of prolonged immobility?
A. * bone demineralization
B. increased respiratory excursion
C. decreased cardiac workload
D. increased peristalsis
E. No correct answer
162.
The nurse is assessing a client who has been immobilized for several weeks. What may
occur as a result of prolonged immobility?
A. * bone demineralization
B. increased respiratory excursion
C. decreased cardiac workload
D. increased peristalsis
E. No correct answer
163.
The nurse is caring for a client who is living at home. The client's family does
everything for the client and does not encourage the client to function independently. This
condition is described as
A. well-adjusted dependence
B. * well-adjusted independence
C. maladjusted dependence
D. codependence
E. No correct answer
164.
The nurse is caring for a client who is living at home. The client's family does
everything for the client and does not encourage the client to function independently. This
condition is described as
A. well-adjusted dependence
B. * well-adjusted independence
C. maladjusted dependence
D. codependence
E. No correct answer
165.
The nurse is lifting a heavy object. Which action, if performed, is incorrect?
A. Keep the feet apart.
B. Pull the object, do not push.
C. * Bend at the knees.
D. Lift with the leg muscles.
E. No correct answer
166.
The nurse is lifting a heavy object. Which action, if performed, is incorrect?
A. Keep the feet apart.
B. Pull the object, do not push.
C. * Bend at the knees.
D. Lift with the leg muscles.
E. No correct answer
167.
The nurse is teaching a client how to walk with crutches using a three-point gait. Which
instruction is correct?
A. move the right crutch forward, then the left foot, followed by the left crutch, and finally
the right foot
B. move the right crutch and the left foot forward together and then the left crutch and the
right foot forward together
C. * move the crutches and weak leg forward together, then move the weight-bearing leg
forward
D. move the crutches forward, then swing the legs forward together
E. No correct answer
168.
The nurse is teaching a client how to walk with crutches using a three-point gait. Which
instruction is correct?
A. move the right crutch forward, then the left foot, followed by the left crutch, and finally
the right foot
B. move the right crutch and the left foot forward together and then the left crutch and the
right foot forward together
C. * move the crutches and weak leg forward together, then move the weight-bearing leg
forward
D. move the crutches forward, then swing the legs forward together
E. No correct answer
169.
The nurse is to ambulate a client who has an IV in his arm, an indwelling urinary
catheter, and a chest tube. What action is appropriate for the nurse to take?
A. Hold the IV so it is higher than the client's heart while ambulating.
B. * Empty the urinary drainage tube before ambulating.
C. Empty the closed chest drainage system before ambulating.
D. Hold the closed chest drainage system above the client's chest level while ambulating.
E. No correct answer
170.
The nurse is to ambulate a client who has an IV in his arm, an indwelling urinary
catheter, and a chest tube. What action is appropriate for the nurse to take?
A. Hold the IV so it is higher than the client's heart while ambulating.
B. * Empty the urinary drainage tube before ambulating.
C. Empty the closed chest drainage system before ambulating.
D. Hold the closed chest drainage system above the client's chest level while ambulating.
E. No correct answer
171.
The nurse making a home visit observes all of the following. Which is a safety hazard
for the client with difficulty in mobility?
A. wall-to-wall carpet in the bedroom
B. curio cabinet filled with mementos
C. * scatter rugs on the kitchen floor
D. night lights in the hall and bathroom
E. No correct answer
172.
The nurse making a home visit observes all of the following. Which is a safety hazard
for the client with difficulty in mobility?
A. wall-to-wall carpet in the bedroom
B. curio cabinet filled with mementos
C. * scatter rugs on the kitchen floor
D. night lights in the hall and bathroom
E. No correct answer
173.
The nurse should measure the patient temperature:
A. In the morning
B. In the evening
C. In the morning and in the evening
D. Every 3 hours
E. Once per week
174.
The nurse should measure the patient temperature:
A. * In the morning and in the evening
B. In the morning
C. In the evening
D. Once per week
E. Every 3 hours
175.
The nurse should write down the patient temperature in the:
A. Case history
B. Statistic talon
C. Ambulatory journal
D. Temperature chart
E. All are possible
176.
The nurse should write down the patient temperature in the:
A. * Temperature chart
B. Case history
C. Statistic talon
D. Ambulatory journal
E. All are possible
177.
The nurse suspects that the client is a substance abuser. Which clinical manifestation
causes the nurse to suspect cocaine abuse rather barbiturates?
A. Shallow respirations
B. Pupillary constriction
C. * Tachycardia
D. Flushing
E. No correct answer
178.
?The nurse understands that communication is a vital component of the human
experience. People communicate in order to
A. express their wishes
B. respond to others
C. * define themselves and find meaning in their interactions with the world around them
D. voice their opinions
E. No correct answer
179.
?The nurse understands that communication is a vital component of the human
experience. People communicate in order to
A. express their wishes
B. respond to others
C. * define themselves and find meaning in their interactions with the world around them
D. voice their opinions
E. No correct answer
180.
The patient blood pressure is 180/100 mm Hg. Assess this data:
A. Normal
B. Arterial hypotension
C. * Arterial hypertension
D. Hyperthermia
E. Bradycardia
181.
The patient can't move and rotate in bad. You must change his position to prevent the
bedsores. What is the turning schedule in this case?
A. every 10 hours
B. every 18 hours
C. every 6 hours
D. every 4 hours
E. every 2 hours
182.
The patient with pneumonia presents with high temperature of 40,0?C, dyspnea, cough,
headache. What should the nurse do the first:
A. Give vitamins
B. Give the drugs to reduce the cough
C. Talk with patient about his condition
D. This patient don’t need any help
E. Decrease the body temperature
183.
The patient with severe pathology has the following injury on his elbows: epidermis is
broken, superficial lesion, no measurable depth; partial-thickness skin loss. What stage of
pressure ulcers does the patient have?
A. 1
B. 3
C. 4
D. 5
E. 2
184.
The patient with severe pathology has the following injury on his shoulder blades: fullthickness skin loss extending into supportive structures, such as muscle, tendon, and bone.
What stage of pressure ulcers does the patient have?
A. 1
B. 2
C. 3
D. 4
E. 5
185.
The patients who received treatment with the pozahospitalnoyi bilateral pneumonia
remains high fever 39 ° C, AT-90/50 mm Hg, CHDR-40/min., Show lysya anemia, oliguria.
Patient transferred to resuscitation department. Priority governmental emergency measure will:
A. * Respiratory care
B. Adequate antibiotics
C. Forced diuresis
D. Introduction reopoliglyukinu
E. Hemosorbtion
186.
The result of temperature measurement of a patient is determined after:
A. 3-5-minute after beginning
B. 5-7-minute after beginning
C. 6-8-minute after beginning
D. 12-15 minute after beginning.
E. 7-10-minute after beginning
187.
The signs of the 1st stage of the pressure ulcer are
A. full-thickness skin loss extending into supportive structures, such as muscle, tendon,
and bone; may undermine and have various sinus tracts
B. full-thickness skin loss down through the dermis; may include subcutaneous tissue; may
undermine adjacent skin
C. epidermis is broken, superficial lesion, no measurable depth. Partial-thickness skin loss
D. no visible signs
E. nonblanchable erythema, redness that remains present over an area under pressure 30
minutes after pressure source is removed. Epidermis remains intact
188.
The temperature of hot water bottle must be
A. 35,5-40 °C
B. * 40,5-46 °C
C. 46,5-50 °C
D. 50,5-55 °C
E. 55,5-60 °C
189.
The temperature of the cold bag must be
A. 5 °C
B. 10 °C
C. * 15 °C
D. 1 °C
E. 7 °C
190.
The term for letting clients express their feelings is
A. * catharsis
B. rapport
C. empathy
D. empowerment
E. No correct answer
191.
The term for letting clients express their feelings is
A. * catharsis
B. rapport
C. empathy
D. empowerment
E. No correct answer
192.
The traditional sites for measuring the body temperature are:
A. rectal, nasal, oral
B. rectal, nasal, axillary
C. axillary, nasal, oral
D. rectal, nasal, intestinal
E. rectal, axillary, oral
193.
The traditional sites for measuring the body temperature are:
A. rectal, nasal, oral
B. rectal, nasal, axillary
C. axillary, nasal, oral
D. rectal, nasal, intestinal
E. * higher than axillary
194.
The victim with severe trauma combined stated apnea, increased pupil, absence of pulse
in the main vessels. What is not Heine measures should apply in asystole?
A. * Artificial respiration, closed heart massage
B. Intravenous hydrocarbonate sodium
C. Introduction of corticosteroids
D. Infusion poliglyucine
E. Introduction V-adrenoblocker
195.
Therapeutic use of self involves what type of skills by the nurse?
A. verbal communication
B. nonverbal communication
C. * both verbal and nonverbal communication
D. primarily technical skills
E. No correct answer
196.
Therapeutic use of self involves what type of skills by the nurse?
A. verbal communication
B. nonverbal communication
C. * both verbal and nonverbal communication
D. primarily technical skills
E. No correct answer
197.
To take a temperature with a mercury-in-glass thermometer, shake the thermometer to
move the mercury down below:
A. 92 degree F
B. 93 degree F
C. 94 degree F
D. 95 degree F
E. 96 degree F
198.
To take a temperature with a mercury-in-glass thermometer, shake the thermometer to
move the mercury down below:
A. 92 degree F
B. 93 degree F
C. 94 degree F
D. 95 degree F
E. * 96 degree F
199.
?to the terminal states belong?
A. * preagony, terminal pause, agony, Clinical Death
B. Agony, clinical and biological death
C. commas agony, Clinical Death
D. preagony, agony, clinical, social death
E. AT decrease to 80 mm Hg, commas, Clinical Death
200.
Token effectively artificial ventilation are:
A. myosis
B. noise and air insufflation during expiration
C. * silent chest excursion
D. protrusion epigastric area during insufflation
E. «Dizziness» of reanimator
201.
Truly caring for what happens to another person is called
A. acceptance
B. * empathy
C. compassion
D. sympathy
E. No correct answer
202.
Truly caring for what happens to another person is called
A. acceptance
B. * empathy
C. compassion
D. sympathy
E. No correct answer
203.
Tuberculin skin test is used to identify:
A. sudden death
B. cancer
C. hypertension
D. * tuberculosis
E. Pneumonia
204.
Understanding another person's perception of a situation is called
A. * empathy
B. acceptance
C. empowerment
D. advocacy
E. No correct answer
205.
Understanding another person's perception of a situation is called
A. * empathy
B.
C.
D.
E.
206.
A.
B.
C.
D.
E.
207.
A.
B.
C.
D.
E.
208.
A.
B.
C.
D.
E.
209.
A.
B.
C.
D.
E.
210.
A.
B.
C.
D.
E.
211.
A.
B.
C.
D.
E.
212.
A.
B.
C.
D.
E.
213.
A.
B.
C.
D.
E.
acceptance
empowerment
advocacy
No correct answer
Vital signs include following:
pulse
temperature
respiratory rate
blood pressure
* all mentioned
What conditions can prolong the time of brain surviving without oxygen?
hyperthermia
dehydration
hypothermia
hypoglycemia
hyperhydration
What equipment is needed for CPR?
one-way breathing mask
plastic eye-glasses
hospital gown
intubation tube
vinyl gloves
What factor(s) can change the body temperature?
infection
temperature of environment
thyroid hormone
age
* all above
What factor(s) can change the body temperature?
infection
temperature of environment
thyroid hormone
age
all above
What is indication for administering cleansing enema:
Diarrhea
Constipation
Preparation for abdominal surgery
Poisoning
All above
What is optimal air temperature for the ward in the therapeutic department?
16-17 ?C
24-25 ?C
18-20 ?C
36,6 ?C
10-15 ?C
What is the appropriate depth of chest compressions during CPR (for adults)?
6-9 cm
3-4 cm
4-6 cm
2-4 cm
7-10 cm
214.
A.
B.
C.
D.
E.
215.
A.
B.
C.
D.
E.
216.
A.
B.
C.
D.
E.
217.
A.
B.
C.
D.
E.
218.
A.
B.
C.
D.
E.
219.
A.
B.
C.
D.
E.
220.
A.
B.
C.
D.
E.
221.
A.
B.
C.
D.
E.
222.
A.
B.
C.
?What is the appropriate order for checking of vital signs before the beginning of CPR?
circulation, airways, breathing
breathing, circulation, airways
airways, breathing, circulation
airways, circulation, breathing
breathing, airways, circulation
What is the appropriate rate of chest compressions during CPR?
40 /min
50 /min
60 /min
70 /min
80 /min
What is the contraindication for administering enema:
Diarrhea
Vomiting
High temperature
Rectal bleeding
All above
What is the correct place for the compressions during CPR?
upper 1/3 of sternum
middle 1/3 of sternum
epigastric area
umbilical area
lower 1/3 of sternum
?What is the main contraindication for enteral tube feeding?
diffused peritonitis
intestinal obstruction that prohibits normal bowel functioning
intractable vomiting; paralytic ileus
severe diarrhea
all listed above
What is the most common risk factor associated with the pressure ulcer development?
drugs usage
smoking
heredity
immobility
drinking
What is the normal body temperature:
97,9 ?F
38,2 ?F
36.6 ?F
68,4 ?F
87,2 ?F
What is the normal body temperature:
* 36,6 °C
38,2 °C
41,0 °C
35,1 °С
98,6 °C
What is the normal body temperature:
87,2 °F
38,2 °F
36,6 °F
D. 68,4 °F
E. * 97,9 °F
223.
What is the normal pulse rate in the adult person:
A. * 60-90 per minute
B. 40-50 per minute
C. 100-120 per minute
D. 90-100 per minute
E. 30-70 per minute
224.
What is the patient position for administering enema:
A. Lying on the back
B. Lying on the right side
C. Lying on the left side
D. Staying
E. Sitting
225.
What is the patient position for using a bedpan:
A. Lying face down
B. Sitting
C. Lying on the left side
D. Staying
E. Lying on the back
226.
What is the primary purpose of putting high-top tennis shoes on the immobilized client
who is positioned in Fowler's and supine position?
A. * to help prevent foot drop
B. to promote comfort
C. to prevent decubitus formation on the heels
D. to aid in turning the client
E. No correct answer
227.
What is the primary purpose of putting high-top tennis shoes on the immobilized client
who is positioned in Fowler's and supine position?
A. * to help prevent foot drop
B. to promote comfort
C. to prevent decubitus formation on the heels
D. to aid in turning the client
E. No correct answer
228.
What is the pulse deficit?
A. * The difference between the rate of heart contractions and distal pulse rate
B. The sum of the rate of heart contractions and pulse rate
C. The difference between the pulse rate and the rate of heart contractions
D. The difference between the pulse rate and the rate of breath
E. The sum of the pulse rate and the rate of breath
229.
What is the right place for the checking of presence of pulse for assessment of
unconscious patient?
A. femoral artery
B. carotid artery
C. clavicular artery
D. brachial artery
E. radial artery
230.
What is the simplest and the most often used route of enteral tube feeding?
A. nasoenteral route
B. gastrostomy route
C. jejunostomy route
D. all listed above
E. esophagostomy route
231.
What is the water temperature for cleansing enema?
A. 10 ?C
B. 50 ?C
C. 100 ?C
D. 80 ?C
E. 37 ?C
232.
What patient position is called “orthopnoe”:
A. * Sitting
B. Lying on the back
C. Lying face down
D. Lying on the left side
E. Lying on the right side
233.
What patient position is used in collapse with loss of consciousness:
A. * Lying on the back with elevated legs
B. Lying on the back with elevated head
C. Sitting
D. Lying on the left side
E. Standing
234.
What recommendation is not used for prevention of pressure ulcers in a patient in
coma?
A. Change child’s position not less than 1 time per 3 hours
B. Keep clothing and linen clean and dry
C. Maintain meticulous skin clearness
D. Ultraviolet radiation (UVR) on the problem parts of skin
E. Help with surgical “cleaning” of affected place, then cover the wound with sterile
dressing soaked with 1 % solution of potassium permanganates; change the dressing 2-3
times per day
235.
What should the nurse do before giving the bedpan to the patient?
A. Rinse it with 10% solution of chloride of lime
B. Rinse it with 10% solution of chloramine
C. Rinse it with 10% solution of hydrogen peroxide
D. Rinse it with warm water
E. Rinse it with cold water
236.
What should the nurse do before giving the bedpan to the patient?
A. Rinse it with 10% % solution of chloride of lime
B. Rinse it with 10% % solution of chloramine
C. Rinse it with 10% % solution of hydrogen peroxide
D. Rinse it with cold water
E. * Rinse it with warm water
237.
What site can be used for body temperature measuring?
A. stomach
B. vagina
C. oral cavity
D. all above
E. chest surface
238.
What site can be used for body temperature measuring?
A. * oral cavity
B. stomach
C. vagina
D. chest surface
E. all above
239.
What solution should the nurse use for disinfection of thermometer after measuring
temperature?
A. 5% solution of chloramines
B. * 2% solution of chloramine
C. 5% solution of chloride of lime
D. 10% solution of sodium chloride
E. 2% solution of ethyl alcohol
240.
What types of thermometers can be used for measuring the body temperature?
A. mercury in glass
B. disposable single-use chemical strip
C. tympanic membrane thermometer
D. electronic
E. all mentioned
241.
What types of thermometers can be used for measuring the body temperature?
A. mercury in glass
B. disposable single-use chemical strip
C. tympanic membrane thermometer
D. electronic
E. * all mentioned
242.
When analyzing components of the communication process, the nurse identifies which
as an internal stimuli?
A. sight
B. sensation
C. taste
D. * hunger
E. No correct answer
243.
When analyzing components of the communication process, the nurse identifies which
as an internal stimuli?
A. sight
B. sensation
C. taste
D. * hunger
E. No correct answer
244.
When assessing a client's musculoskeletal status,
A. * warn the client when there may be discomfort
B. stretch all joints to their maximum limit
C. know that a 0 rating of muscle tone means weakness
D. know that a 0 rating on flexibility means total inflexibility
E. No correct answer
245.
When assessing a client's musculoskeletal status,
A. * warn the client when there may be discomfort
B. stretch all joints to their maximum limit
C. know that a 0 rating of muscle tone means weakness
D. know that a 0 rating on flexibility means total inflexibility
E. No correct answer
246.
When interviewing a client, there are long pauses of silence. The client does not respond
to the nurse's questions. All of the following are appropriate responses by the nurse to the
clients silence EXCEPT
A. sitting quietly and observing the client's behavior
B. controlling one's own discomfort during quiet periods or conversational lulls
C. using appropriate eye contact
D. * asking the patient to answer the question at hand
E. No correct answer
247.
When interviewing a client, there are long pauses of silence. The client does not respond
to the nurse's questions. All of the following are appropriate responses by the nurse to the
clients silence EXCEPT
A. sitting quietly and observing the client's behavior
B. controlling one's own discomfort during quiet periods or conversational lulls
C. using appropriate eye contact
D. * asking the patient to answer the question at hand
E. No correct answer
248.
When transferring a client from a bed to a wheelchair, the nurse should
A. raise the height of the bed
B. position the wheelchair 2 feet from the side of the bed
C. * lock the wheelchair brakes and elevate the foot pedals
D. pivot the client so the client is facing the wheelchair
E. No correct answer
249.
When transferring a client from a bed to a wheelchair, the nurse should
A. raise the height of the bed
B. position the wheelchair 2 feet from the side of the bed
C. * lock the wheelchair brakes and elevate the foot pedals
D. pivot the client so the client is facing the wheelchair
E. No correct answer
250.
When transferring a client from the bed to a chair, the nurse should do all of the
following EXCEPT
A. remove her stethoscope before transfer
B. * ask the client to grab onto her neck
C. place the wheelchair parallel to the bed and as close as possible
D. place her thumbs downward to prevent potential wrist injury as she lifts
E. No correct answer
251.
When transferring a client from the bed to a chair, the nurse should do all of the
following EXCEPT
A. remove her stethoscope before transfer
B. * ask the client to grab onto her neck
C. place the wheelchair parallel to the bed and as close as possible
D. place her thumbs downward to prevent potential wrist injury as she lifts
E. No correct answer
252.
When working with clients and families under severe or chronic stress, the nurse should
A. avoid interaction with the individuals until the stress level is normalized
B. provide communication via written form only
C. * speak in simple terms repeating information as needed
D. obtain medications for these individuals to take to decrease their anxiety before any
interaction with them
E. No correct answer
253.
When working with clients and families under severe or chronic stress, the nurse should
A. avoid interaction with the individuals until the stress level is normalized
B. provide communication via written form only
C. * speak in simple terms repeating information as needed
D. obtain medications for these individuals to take to decrease their anxiety before any
interaction with them
E. No correct answer
254.
Which activity is most likely to take place in the space known as social distance?
A. checking the client's IV
B. * rocking a baby
C. leading a group discussion
D. assessing the client's pain
E. No correct answer
255.
Which activity is most likely to take place in the space known as social distance?
A. checking the client's IV
B. * rocking a baby
C. leading a group discussion
D. assessing the client's pain
E. No correct answer
256.
Which characteristic response to stimulant drugs increases their potential for abuse?
A. They are not habit-forming.
B. They have sedating qualities.
C. * They increase general metabolism
D. Their use is not controlled by the DEA.
E. No correct answer
257.
Which characteristic, action, or behavior fails to meet the criteria for the nursing
diagnosis of substance abuse?
A. * The client uses the substance daily.
B. The client's behavior when using the substance is not socially acceptable.
C. The client continues to use the substance even though she or he has expressed a desire
to stop.
D. The client continues to use the substance even though it causes him or her to have
chronic hypertension.
E. No correct answer
258.
Which comment is appropriate for the nurse to make during the introduction stage of the
assessment interview?
A. "It is almost time for me to leave. Do you have any questions for me?"
B. "Describe your pain."
C. * "I need to ask you a few questions about your health so we can better plan your care."
D. "Describe the number and characteristics of your bowel movements."
E. No correct answer
259.
Which comment is appropriate for the nurse to make during the introduction stage of the
assessment interview?
A. "It is almost time for me to leave. Do you have any questions for me?"
B. "Describe your pain."
C. * "I need to ask you a few questions about your health so we can better plan your care."
D. "Describe the number and characteristics of your bowel movements."
E. No correct answer
260.
Which is an example of a closed question?
A. "What is the reason for your visit today?"
B. "Tell me about your family."
C. * "What is your date of birth?"
D. "What type of food do you prefer?"
E. No correct answer
261.
Which is an example of a closed question?
A. "What is the reason for your visit today?"
B. "Tell me about your family."
C. * "What is your date of birth?"
D. "What type of food do you prefer?"
E. No correct answer
262.
Which is the first step in the guidelines for promoting a caring interview with a patient?
A. observe
B.
C.
D.
E.
263.
A.
B.
C.
D.
E.
264.
A.
B.
C.
D.
E.
265.
A.
B.
C.
D.
E.
266.
A.
B.
C.
D.
E.
267.
A.
B.
C.
D.
E.
268.
A.
B.
C.
D.
E.
269.
A.
B.
C.
D.
E.
270.
A.
B.
C.
D.
E.
establish rapport
* listen attentively
gather information
No correct answer
Which is the first step in the guidelines for promoting a caring interview with a patient?
observe
establish rapport
* listen attentively
gather information
No correct answer
Which method of temperature measuring is the safest:
oral
axillary
measuring pulmonary artery temperature
measuring ear canal temperature
rectal
Which method of temperature measuring is the safest:
oral
* axillary
rectal
measuring pulmonary artery temperature
measuring ear canal temperature
Which nursing diagnosis is appropriate for a woman who abuses anabolic steroids?
Risk for Injury related to decreased muscle coordination
Hypothermia related to decreased metabolic rate
Chronic Confusion related to sodium and water retention
* Disturbed Body Image related to presence of facial hair
No correct answer
Which of the following helps prevent decubitus ulcers?
bed rest
liquid diet
support hose
back rubs
none of them
Which of the following is an example of subjective data?
* nausea
cyanosis
pulse 62
facial grimace
No correct answer
Which of the following is an example of subjective data?
* nausea
cyanosis
pulse 62
facial grimace
No correct answer
Which of the following is considered a primary source of data?
results of diagnostic tests
family
medical records
* client
No correct answer
271.
Which of the following is considered a primary source of data?
A. results of diagnostic tests
B. family
C. medical records
D. * client
E. No correct answer
272.
Which of the following is not an aerobic exercise?
A. rowing
B. bicep curls
C. * walking
D. jumping rope
E. No correct answer
273.
Which of the following is not an aerobic exercise?
A. rowing
B. bicep curls
C. * walking
D. jumping rope
E. No correct answer
274.
?Which of the following is not included in the assessment step of the nursing process?
A. data collection
B. data documentation
C. data verification
D. * data evaluation
E. No correct answer
275.
?Which of the following is not included in the assessment step of the nursing process?
A. data collection
B. data documentation
C. data verification
D. * data evaluation
E. No correct answer
276.
Which of the following nursing actions is most likely to take place in the space known
as personal distance?
A. taking vital signs
B. one-on-one teaching
C. * teaching a class
D. giving an injection
E. No correct answer
277.
Which of the following nursing actions is most likely to take place in the space known
as personal distance?
A. taking vital signs
B. one-on-one teaching
C. * teaching a class
D. giving an injection
E. No correct answer
278.
Which of the special equipment devices is not primarily used to prevent skin
breakdown?
A. air-filled mattress
B. sheepskin
C. egg crate mattress
D. * hand-wrist splints
E. No correct answer
279.
Which of the special equipment devices is not primarily used to prevent skin
breakdown?
A. air-filled mattress
B. sheepskin
C. egg crate mattress
D. * hand-wrist splints
E. No correct answer
280.
Which of these signs are not the major signs of clinical death?
A. * lack of blood pressure
B. absence of pulsation on the femoral artery
C. lack of breathing
D. midriaz, pupil not responding to light
E. absence of pulsation on the carotid artery
281.
Which patients require frequent monitoring of body temperature?
A. pregnant women
B. newborn
C. older adults
D. postoperative patients
E. all above
282.
Which patients require frequent monitoring of body temperature?
A. * postoperative patients
B. pregnant women
C. newborn
D. older adults
E. all above
283.
Which problem or manifestation in a 125-pound, 40-year-old woman, 1-day
postoperative for a total abdominal hysterectomy, leads the nurse to suspect possible substance
abuse?
A. She has vomited nine times during the first 24 hours after surgery.
B. * Morphine 15 mg (subcutaneous) has failed to relieve her pain.
C. She has been unable to void after removal of the Foley.
D. Her wound drainage is greater than expected.
E. No correct answer
284.
Which solution can be used for bath disinfection after taking a bath at the hospital?
A. 0,5% solution of hexachlorane
B. 0,5% solution of chloride of lime
C. solution of manganese permanganate
D. solution of hydrogen peroxide
E. 2% solution of chloramines
285.
Which solution can be used for bath disinfection after taking a bath at the hospital?
A. * 0,5% solution of chloride of lime
B. 0,5% solution of hexachlorane
C. 2% solution of chloramines
D. solution of manganese permanganate
E. solution of hydrogen peroxide
286.
Which statement is correct about falls? Most falls
A. occur during the day
B. involve wheelchairs
C. * occur in the kitchen or bathroom
D. involve clients with poor hearing
E. No correct answer
287.
Which statement is correct about falls? Most falls
A.
B.
C.
D.
E.
occur during the day
involve wheelchairs
* occur in the kitchen or bathroom
involve clients with poor hearing
No correct answer
288.
Which statement is not correct regarding performing passive range of motion exercises
on a client?
A. * The nurse should start at the client's foot and move to the head.
B. The head should be rotated 1/4 turn from side to side and then flexed and extended.
C. The arms should be flexed and extended and adducted and abducted.
D. Do not flex, extend, rotate, abduct, or adduct a joint if the client complains of
discomfort.
E. No correct answer
289.
Which statement is not correct regarding performing passive range of motion exercises
on a client?
A. * The nurse should start at the client's foot and move to the head.
B. The head should be rotated 1/4 turn from side to side and then flexed and extended.
C. The arms should be flexed and extended and adducted and abducted.
D. Do not flex, extend, rotate, abduct, or adduct a joint if the client complains of
discomfort.
E. No correct answer
290.
Which statement is true about the cognitive part of communication?
A. * The receiver interprets and gives meaning to stimuli.
B. It is the medium through which a message is transmitted.
C. It refers to spoken words and cues.
D. It relates to the distance between persons.
E. No correct answer
291.
Which statement is true about the cognitive part of communication?
A. * The receiver interprets and gives meaning to stimuli.
B. It is the medium through which a message is transmitted.
C. It refers to spoken words and cues.
D. It relates to the distance between persons.
E. No correct answer
292.
Which statement or information obtained from a client during assessment for alcohol
abuse alerts the nurse to the possibility of alcohol addiction?
A. The client says he or she drinks alcohol to feel less stressed and have a good time.
B. The client has been arrested once for driving under the influence of alcohol.
C. * The client uses alcohol to stop his or her hands from shaking.
D. The client drinks alcohol daily.
E. No correct answer
293.
Which statement regarding substance abuse is true?
A. Substance abuse should not be considered a public health problem because it only
affects individuals rather than society as a whole.
B. * Even when a person is not “addicted” to a substance, abuse can cause physical,
psychological, and social problems.
C. Substance abuse is rarely seen among middle-class clients.
D. Cocaine is the most commonly abused substance.
E. No correct answer
294.
While carrying a patient by stretcher to his bed, we should observe such rules:
A. The head end of the stretcher must be situated athward to head end of the bad
B. The head end of the stretcher must be situated athward to the middle of the bad.
C. The stretcher must be parallel to the bed
D. Stretchers is placed parallel a bed in the distance 1m from the bed.
E. The leg end of stretchers is placed athwart to the leg end of the bed.
295.
While putting a patient from stretchers to his bed:
A. The medical staff should first insert their hands under the patient head and neck; second
- under the back of patient; third - under thighs and shins of patient
B. First assistant - insert their hands under the head and shoulder of patient; second
assistant - under buttocks; third assistant - under knee bends and shins of patient.
C. First assistant - insert their hands under a head and shoulder of patient; second assistant
- under a trunk and buttocks; third assistant - under knee bends and shins of patient.
D. First - insert their hands under head and shoulder of patient; second - under a trunk and
buttocks; third - under shins of patient.
E. First assistant - insert their hands under the head and shoulder of patient; second
assistant - under a trunk and buttocks; third assistant - under thighs and shins of patient
296.
Why is it important to identify a substance abuser in the medical-surgical acute care
setting?
A. To enable the staff to take appropriate actions to protect themselves and other clients
B. * To anticipate additional care needs necessitated by withdrawal
C. To avoid being “taken in” by a manipulative client
D. To inform local authorities about illegal behavior
E. No correct answer
297.
Worker was electrocuted a few minutes ago. he lost consciousness. Had seizures.
Effects of the electric current was discontinued. patient is laying down, no breathing, no pulse
is determined, the skin cyonatic, pupil dilated, do not react to light. What first aid should be
provided?
A. Introduction anesthetic agents
B. Introduction of anticonvulsants
C. Intravenous reopoliglyukine
D. * Closed heart massage and artificial ventilation of lungs
E. Intravenous breathing analeptic
298.
You are teaching a client why immobility has so many negative effects on the body.
When talking about the effects of immobility on the gastrointestinal system, you would include
all of the following EXCEPT
A. fecal impaction
B. * stress ulcers
C. diarrhea
D. decreased appetite
E. No correct answer
299.
You are teaching a client why immobility has so many negative effects on the body.
When talking about the effects of immobility on the gastrointestinal system, you would include
all of the following EXCEPT
A. fecal impaction
B. * stress ulcers
C. diarrhea
D. decreased appetite
E. No correct answer
300.
You are the charge nurse. You have been asked to meet with the husband of a client
being cared for on the unit. The staff tells you he is very angry and wants to talk to someone
about the lousy care his wife is receiving. Which of the following communication techniques
should you use when interacting with this individual?
A. Arrange the setting so the husband is in front of the door with you directly behind him.
B. Start the conversation with a handshake or touch to his shoulder to express your care
and concern.
C. * Do not turn your back on the man.
D. Stay above his line of vision.
E. No correct answer
301.
You are the charge nurse. You have been asked to meet with the husband of a client
being cared for on the unit. The staff tells you he is very angry and wants to talk to someone
about the lousy care his wife is receiving. Which of the following communication techniques
should you use when interacting with this individual?
A. Arrange the setting so the husband is in front of the door with you directly behind him.
B. Start the conversation with a handshake or touch to his shoulder to express your care
and concern.
C. * Do not turn your back on the man.
D. Stay above his line of vision.
E. No correct answer
302.
young girl is pulled from a lake in 3 minutes after drowning. Unconscious. Not
breathing, gray foam is coming from the mouth. Skin violet-cyonotic color. Pulse of carotid
can’t be determined. Specify the procedure for granting reanimation assistance.
A. * Clean your mouth and throat finger hold CPR "mouth to mouth, closed cardiac
massage;
B. Rotate the victim head-down, click on December stand for the withdrawal of water,
hold CPR "mouth to mouth;
C. Immediately conduct CPR closed covered heart massage;
D. clear water from the airways, make CPR by Silvester;
E. Immediately call the EMT team, before they arrive make a CPR for Silvester and closed
cardiac massage.
303.
What is the right place for the checking of presence of pulse for assessment of
unconscious patient?
A. femoral artery
B. radial artery
C. *carotid artery
D. clavicular artery
E. brachial artery
304.
How many seconds do you have to check for the presence of a pulse?
A. 10-20 sec
B. *5-10 sec
C. 1-10 sec
D. 15-30 sec
E. 20-25 sec
305.
What is the right place for teh checking of pulse presense for assessment of
unconscious patient ?
A. radial artery
B. *carotid artery
C. clavicular artery
D. brachial artery
E. femoral artery
306.
What is the appropriate depth of chest compressions during CPR (for adults)?
A. 6-9 cm
B. 3-4 cm
C. 7-10 cm
D. 2-4 cm
E. *4-6 cm
307.
What is the appropriate rate of chest compressions during CPR?
A. 40 /min
B.
C.
D.
E.
50 /min
60 /min
70 /min
*80 /min
308.
What equipment is needed for CPR?
A. *one-way breathing mask
B. vinyl gloves
C. plastic eye-glasses
D. hospital gown
E. intubation tube
309.
How many minutes can the brain survive without oxygen in normal conditions?
A. 3-5 min
B. * 4-6 min
C. 1-3 min
D. 5-8 min
E. 7-10 min
310.
How many rescue breaths per minute do you have to give for the victim if he has a pulse
but is not breathing spontaneously?
A. 7
B. *10
C. 15
D. 20
E. 1
311.
During CPR you must check the presence of breathing and pulse every
A. 1 min
B. 5 min
C. 10 min
D. 7 min
E. *3 min
312.
What conditions can prolong the time of brain surviving without oxygen?
A. * hypothermia
B. Hyperthermia
C. Dehydration
D. Hyperhydration
E. hypoglycemia
313.
How many stages does the pressure ulcer (bed sore) have?
A. 2
B. 3
C. *4
D. 5
Situation tasks
1. The signs of the 1st stage of the pressure ulcer are
A. full-thickness skin loss extending into supportive structures, such as muscle, tendon,
and bone; may undermine and have various sinus tracts
B. full-thickness skin loss down through the dermis; may include subcutaneous tissue; may
undermine adjacent skin
2.
3.
4.
5.
6.
7.
8.
C. *nonblanchable erythema, redness that remains present over an area under pressure 30
minutes after pressure source is removed. Epidermis remains intact
D. epidermis is broken, superficial lesion, no measurable depth. Partial-thickness skin loss
E. no visible signs
The patient with severe pathology has the following injury on his elbows: epidermis is broken,
superficial lesion, no measurable depth; partial-thickness skin loss. What stage of pressure
ulcers does the patient have?
A. 1
B. *2
C. 3
D. 4
E. 5
The signs of the 3rd stage of the pressure ulcer are
A. no visible signs
B. nonblanchable erythema, redness that remains present over an area under pressure 30
minutes after pressure source is removed. Epidermis remains intact
C. epidermis is broken, superficial lesion, no measurable depth. Partial-thickness skin loss
D. full-thickness skin loss extending into supportive structures, such as muscle, tendon,
and bone; may undermine and have various sinus tracts
E. *full-thickness skin loss down through the dermis; may include subcutaneous tissue;
may undermine adjacent skin
The patient with severe pathology has the following injury on his shoulder blades: fullthickness skin loss extending into supportive structures, such as muscle, tendon, and bone.
What stage of pressure ulcers does the patient have?
A. 1
B. 2
C. 3
D. *4
E. 5
Equipment used for prevention of the pressure ulcers
A. pillows, rolled-up blankets or towels
B. egg-crate mattress
C. specialty beds, heel and elbow protectors
D. lotion or powder, soap and water
E. *all listed above
What is the most common risk factor associated with the pressure ulcer development?
A. drinking
B. smoking
C. heredity
D. immobility
E. *drugs usage
The patient can't move and rotate in bad. You must change his position to prevent the bedsores.
What is the turning schedule in this case?
A. every 10 hours
B. every 18 hours
C. every 6 hours
D. every 4 hours
E. *every 2 hours
What is the main contraindication for enteral tube feeding?
A. diffused peritonitis
B. intestinal obstruction that prohibits normal bowel functioning
C. intractable vomiting; paralytic ileus
D. severe diarrhea
E. *all listed above
9. What is the simplest and the most often used route of enteral tube feeding?
A. esophagostomy route
B. gastrostomy route
C. *nasoenteral route
D. jejunostomy route
E. all listed above
10. How can you determine the required length of a nasogastric tube?
A. *by measuring from bridge of nose to earlobe and to xiphoid process of sternum
B. by measuring from bridge of nose to umbilicus
C. by measuring from bridge of nose to xiphoid process of sternum
D. by measuring from mouth to umbilicus
E. by measuring from earlobe to pubis
11. Decubitus ulcer is caused by:
A. poor circulation
B. pressure on the scin
C. poor nutrition
D. *all of the above
E. none of the above
12. You have to instruct the patient to keep the hot moist compress not longer than
A. 10 min
B. 15 min
C. 20 min
D. 25 min
E. *30 min
13. The temperature of hot water bottle must be
A. 35,5-40 C
B. *40,5-46 C
C. 46,5-50 C
D. 50,5-55 C
E. 55,5-60 C
14. Cold therapy is used to
A. increase blood flow in place
B. provide vasodilatation
C. *provide vasoconstriction
D. decrease the muscle spasms
E. all listed above
15. The patient has a hepatic colic. What will you prescribe to the patient?
A. morphine
B. cold bag on right hypochondrium
C. hot bag on left hypochondrium
D. cold bag on left hypochondrium
E. *hot bag on right hypochondrium
16. During the assessment of the patient you discover the symptoms of gastric bleeding. The best
action would be to use
A. hot water bottle on the epigastric area
B. cold bag on umbilical area
C. hot water bottle on the chest
D. *cold bag on the epigastric area
E. cold bag on the chest
17. The temperature of the cold bag must be
A. 5 C
B. 10 C
C. *15 C
D. 1 C
E. 7 C
18. The duration of the cold treatment is not longer than
A. 10 min
B. 15 min
C. 20 min
D. 25 min
E. *30 min
19. Vital signs include following:
A. pulse
B. temperature
C. respiratory rate
D. blood pressure
E. *all mentioned
20. What factor(s) can change the body temperature?
A. infection
B. temperature of environment
C. thyroid hormone
D. age
E. *all above
21. What condition can prolong the time of brain surviving without oxygen?
A. hyperthermia
B. dehydration
C. *hypothermia
D. hypoglycemia
E. hyperhydration
22. What site can be used for body temperature measuring?
A. *oral cavity
B. stomach
C. vagina
D. chest surface
E. all above
23. The traditional sites for measuring the body temperature are:
A. rectal, nasal, oral
B. *rectal, axillary, oral
C. rectal, nasal, axillary
D. axillary, nasal, oral
E. rectal, nasal, intestinal
24. Oral temperature is:
A. *higher than axillary
B. lower than axillary
C. the same as axillary
D. the same as rectal
E. all are correct
25. Rectal temperature is:
A. *higher than axillary
B. lower than axillary
C. the same as axillary
D. the same as oral
E. lower than oral
26. Nasal temperature measuring is:
A. safe and accurate
B. unsafe and accurate
C. safe and not accurate
D. *not applicable
E. applicable only in patients without teeth
27. Which method of temperature measuring is the safest:
A. oral
B. *axillary
C. rectal
D. measuring pulmonary artery temperature
E. measuring ear canal temperature
28. The highest body temperature is temperature measured in the:
A. axilla
B. *rectum
C. oral cavity
D. ear
E. all are equal
29. Oral route temperature measuring should be delayed if:
A. patient has smoked
B. patient has ingested hot meals
C. patient has ingested cold meals
D. *no reasons for delay
E. all mentioned reasons
30. Before taking a temperature with a mercury-in-glass thermometer, you should shake the
thermometer to move the mercury:
A. *below normal body temperature
B. at the level of normal body temperature
C. at the level of 37 °C
D. at the level of 40 °C
E. at the level of 0 °C
31. To take a temperature with a mercury-in-glass thermometer, shake the thermometer to move
the mercury down below:
A. 92 degree F
B. 93 degree F
C. 94 degree F
D. 95 degree F
E. *96 degree F
32. Conversion of Centigrade to Fahrenheit is
A. Degrees F = 9/5 (°C) + 40
B. Degrees F = 7/5 (°C) + 32
C. *Degrees F = 9/5 (°C) + 32
D. Degrees F = 7/5 (°C) + 52
E. Degrees F = 9/5 (°C) + 45
33. Conversion of Fahrenheit to Celsium is
A. Degrees C = 8/9 (°C - 40)
B. *Degrees C = 5/9 (°C - 32)
C. Degrees C = 8/5 (°C - 32)
D. Degrees C = 3/5 (°C - 43)
E. Degrees C = 9/5 (°C - 52)
34. Which patients require frequent monitoring of body temperature?
A. *postoperative patients
B. pregnant women
C. newborn
D. older adults
E. all above
35. What types of thermometers can be used for measuring the body temperature?
A. mercury in glass
B. disposable single-use chemical strip
C. tympanic membrane thermometer
D. electronic
E. *all mentioned
36. What is the normal body temperature:
A. *36,6 ° C
B. 38,2 °C
C. 41,0 ° C
D. 35,1 ° C
E. 98,6 ° F
37. What is the normal body temperature:
A. 87,2 °F
B. 38,2 °F
C. 36.6 °F
D. 68,4 °F
E. *97,9 °F
38. What solution should the nurse use for disinfection of thermometer after measuring
temperature?
A. *5% solution of chloramines
39. How often should the ward of the therapeutic department be cleaned up?
A. as often as necessary
B. *twice per day
C. every morning
D. every evening
E. it doesn’t need any cleaning
40. What amount of chloramine powder is necessary to prepare 3 liters of 3% chloramine?
A. *90 g
B. 30 g
C. 3 g
D. 60 g
E. 9 g
41. What disinfective solution is used for cleaning?
A. *0,5% solution of chloride of lime
B. 10% solution of chloride of lime
C. 1% solution of chloramines
D. solution of manganese permanganate
E. solution of hydrogen peroxide
42. The patient with a history tuberculosis in the past was taking to the admission department.
What solution can nurse use to disinfect medical instruments, used for this patient
examination?
A. *5 % solution of chloramine for 240 min
B. 3 % solution of chloramine for 60 min
C. 5 % solution of sulfochlorantine for 240 min
D. 1 % solution of chloramine for 60 min
E. 5 % solution of chlorane for 240 min
43. The patient with suspected gastric bleeding was taken to the admission department. He’s
feeling well and can walk. How should this patient be transferred to the surgical department?
A. walking with the nurse to accompany
B. walking with the doctor
C. *on the stretcher
D. walking without any other person to accompany
E. all are possible
44. What is optimal air temperature for the ward in the therapeutic department?
A. 16-17 °C
B. *18-20 °C
C. 24-25 °C
D. 10-15 °C
E. 36,6 °C
45. Which solution can be used for bath disinfection after taking a bath at the hospital?
A. *0,5% solution of chloride of lime
B. 0,5% solution of hexachlorane
C. 2% solution of chloramines
D. solution of manganese permanganate
E. solution of hydrogen peroxide
46. What should the nurse do before giving the bedpan to the patient?
A. Rinse it with 10% % solution of chloride of lime
B. Rinse it with 10% % solution of chloramine
C. Rinse it with 10% % solution of hydrogen peroxide
D. Rinse it with cold water
E. *Rinse it with warm water
47. Increase of body temperature is called:
A. *Hyperthermia
B. Hypothermia
C. Hypertension
D. Arrhythmia
E. Poliuria
48. If body temperature is increased 1 °C, the pulse rate will be:
A. The same as in normal temperature
B. *Increased 8-10 beats
C. Decreased 8-10 beats
D. Increased 20-25 beats
E. Decreased 20 beats
49. What is the pulse deficit?
A. *The difference between the rate of heart contractions and distal pulse rate
B. The sum of the rate of heart contractions and pulse rate
C. The difference between the pulse rate and the rate of heart contractions
D. The difference between the pulse rate and the rate of breath
E. The sum of the pulse rate and the rate of breath
50. The most common location for assessing the pulse is:
A. *Radial artery
B. Brachial artery
C. Femoral artery
D. Over the heart
E. Temporal artery
51. You have to take the blood pressure of the patient. What is the most common location for the
placement of the cuff?
A. On the right forearm
B. On the left forearm
C. *On the middle third of the upper arm
D. On the upper third of upper arm
E. On the hip
52. In assessing the patient's body temperature, it was found to be 38,5 °C. This is:
A. Normal
B. *Elevated
C. Decreased
D. Subfebril
E. All are correct
53. The patient with pneumonia presents with high temperature of 40,0°C, dyspnea, cough,
headache. What should the nurse do the first:
A. *Decrease the body temperature
B. Give vitamins
C. Give the drugs to reduce the cough
D. Talk with patient about his condition
E. This patient don’t need any help
54. The patient blood pressure is 180/100 mm Hg. Assess this data:
A. Normal
B. Arterial hypotension
C. *Arterial hypertension
D. Hyperthermia
E. Bradycardia
55. The nurse should measure the patient temperature:
A. *In the morning and in the evening
B. In the morning
C. In the evening
D. Once per week
E. Every 3 hours
56. The nurse should write down the patient temperature in the:
A. *Temperature chart
B. Case history
C. Statistic talon
D. Ambulatory journal
E. All are possible
57. What patient position is called “orthopnoe”:
A. *Sitting
B. Lying on the back
C. Lying face down
D. Lying on the left side
E. Lying on the right side
58. What patient position is used in asthma
A. *Sitting
B. Lying on the back
C. Lying on the left side
D. Standing
E. Lying on the back with elevated head
59. How many minutes can the brain survive without oxygen in normal conditions?
A. *4-6 minutes
B. 3-5 minutes
C. 1-3 minutes
D. 5-8 minutes
E. 7-10 minutes
60. Which of the following helps prevent decubitus ulcer?
A. *back rubs
B. bed rest
C. liquid diet
D. support hose
E. none of them
61. What is the water temperature for cleansing enema?
A. 10 °C
B. *37 °C
C. 50 °C
D. 100 °C
E. 80 °C
62. What is the patient position for using a bedpan:
A. *Lying on the back
B. Lying face down
C. Sitting
D. Lying on the left side
E. Staying
63. What is the patient position for administering enema:
A. Lying on the back
B. Lying on the right side
C. Sitting
D. *Lying on the left side
E. Staying
64. What is the contraindication for administering enema:
A. *Rectal bleeding
B. Diarrhea
C. Vomiting
D. High temperature
E. All above
65. What is indication for administering cleansing enema:
A. Diarrhea
B. Constipation
C. Preparation for abdominal surgery
D. Poisoning
E. *All above
66. Which patients require daily measuring of the weight?
A. *Patient with edema receiving diuretics
B. Pregnant women
C. Child with a fever
D. Patient with hypertension
E. Patient after surgery
67. Which types of scales can be used for measuring weight:
A. Standing scale
B. Stretcher scale
C. Chair scale
D. Bed scale
E. *All above
68. Pulse rate less than 60 beats per minute in an adult is called:
A. *Bradycardia
B. Tachycardia
C. Arrhythmia
D. Normal pulse rate
E. Hyperthermia
69. Pulse rate more than 100 beats per minute in an adult is called:
A. Bradycardia
B. *Tachycardia
C. Arrhythmia
D. Normal pulse rate
E. Hyperthermia
70. What is pulse rhythm?
A. *The regularity of the heartbeat
B. The pulse rate per minute
C. The pulse rate determined by auscultation
D. The pulse rate per second
E. No correct answer
71. How can you determine the apical pulse?
A. By palpation of carotid artery
B. By palpation of radial artery
C. *By palpation of the point of maximal impulse
D. By auscultation the femoral artery
E. By percussion
72. What equipment is necessary to measure the blood pressure?
A. *the sphygmomanometer and stethoscope
B. the sphygmomanometer, thermometer and stethoscope
C. the syringe and stethoscope
D. stethoscope
E. thermometer
73. What sounds can you hear during measuring the blood pressure?
A. *Korotkoff’s sounds
B. Bowel sounds
C. Pulsation
D. You can’t hear any sounds
E. You can hear all sounds listed above
74. What level of blood pressure is normal for healthy adult?
A. *125/80 mm Hg
B. 80/50 mm Hg
C. 170/100 mm Hg
D. 170/120 mm Hg
E. 200/100 mm Hg
75. Blood pressure screening provides data on risks of:
A. sudden death
B. cancer
C. *hypertension
D. coronary heart disease
E. arrhythmia
76. Tuberculin skin test is used to identify:
A. sudden death
B. cancer
C. hypertension
D. *tuberculosis
E. pneumonia
77. Pediculosis evaluation should be done in:
A. surgical department
B. outpatient dept
C. ambulance
D. *Admission dept
E. at home
78. A word «deontology» in a translation from Greek means:
A. Handicraft, needlework
B. Science is about illnesses and complications of diseases
C. *Science dealing with duty, moral obligation, and right action
D. Science is about medical errors
E. Science about the care of patients
79. Conducting of subjective inspection of a patient, the doctor begins from:
A. Determination receipt of the patient.
B. Review the patient
C. *Determination of complaints of the patient.
D. Conducting of anthropometric researches.
E. Establishment of previous diagnosis
80. Conducting of objective inspection of a patient, the doctor begins from:
A. Conducting of anthropometric researches.
B. Research of pulse.
C. Measuring of arterial pressure
D. To the review of place of disease.
E. *Observing the patient
81. A patient with loss of consiousness due to low blood pressure is delivered to the hospital. How
should you position the patient during his inspection and transporting?
A. Horizontal on the back with lifted on 10-15° a main end of the bad.
B. Horizontal on the back position.
C. Position on stretchers with a head inclined ahead (a chin touched to a pectoral wall).
D. *Supine with the leg of the bed lifted 10-15°.
E. Position is not important.
82. Hygienic- preventive treatment in the hospital is not conducted, if:
A. A patient renounces to conduct treatment
B. a patient with the trophic ulcers of the skin.
C. *a patient in an extremely difficult condition.
D. a patient enters only for conducting of common inspection
E. a patient infected by AIDS.
83. A temperature of water of hygienic bath in the diagnostic department during admission is:
A. 33-34° C
B. *35-36° C.
C. 36-37° C.
D. 38° C
E. 38-39° C
84. A temperature of air (°C) and humidity (%) in the rooms of the hospital is:
A. 16-18°C, 45-50 %.
B. 17-19°C, 45-50 %.
C. 23-24°C, 60-65%
D. 19-21°C, 50-55 %
E. *18-20°C, 50-55 %
85. Bed linen for patients must be changed:
A. Not longer than once every 7 days.
B. Not longer than once every 10 days.
C. *Not longer than once every 12 days.
D. Not longer than once every 14 days.
E. Not longer than once every 15-16 days.
86. Hygienical care of a patient in an admitting department contains such maintenance list of
measures:
A. The special treatment of patient depending on his state and features of disease;
hygienical bath or shower; changing clothes of the patient.
B. Hygienical bath or shower; the special treatment of patient depending on his state and
features of disease; changing clothes of the patient.
C. *Hygienical bath or shower; changing clothes of the patient; the special treatment of
patient depending on his state and features of disease
D. Hygienical bath; changing clothes of the patient
E. The special treatment of the patient depending on his state and features of disease;
changing clothes of the patient
87. There is a patient with the trauma of hand in an admitting department. After hygienic care the
patient should be dressed. What is the first stage of dressing the patient in a shirt?
A. To dress through head.
B. To dress through the healthy hand.
C. *To dress through hand with the trawma
D. To dress through both hands simultaneously.
E. All answers are correct
88. While carrying a patient by stretcher to his bed, we should observe such rules:
A. *The stretcher must be parallel to the bed
B. The head end of the stretcher must be situated athward to head end of the bad
C. The head end of the stretcher must be situated athward to the middle of the bad.
D. The leg end of stretchers is placed athwart to the leg end of the bed.
E. Stretchers is placed parallel a bed in the distance 1m from the bed.
89. While putting a patient from stretchers to his bed:
A. The medical staff should first insert their hands under the patient head and neck; second
- under the back of patient; third - under thighs and shins of patient
B. First assistant - insert their hands under the head and shoulder of patient; second
assistant - under buttocks; third assistant - under knee bends and shins of patient.
C. *First assistant - insert their hands under the head and shoulder of patient; second
assistant - under a trunk and buttocks; third assistant - under thighs and shins of patient
D. First assistant - insert their hands under a head and shoulder of patient; second assistant
- under a trunk and buttocks; third assistant - under knee bends and shins of patient.
E. First - insert their hands under head and shoulder of patient; second - under a trunk and
buttocks; third - under shins of patient.
90. For the cleaning of manipulations rooms and hygienic rooms you should apply:
A. 0,5 % solution of desaktine.
B. *0,2 % solution of desaktine or 0,25% solution of lizoformine-3000.
C. Triple solution
D. 0,25 % solution of chlorhexidine
E. 1% water solution of chloramine.
91. For the number of measures for providing a protective mode we should perform the following
A. Control of adopting medicines
B. Regular ventilation of wards.
C. *Removal of external irritants and negative emotions.
D. Wet cleaning of the departments
E. Prophylaxis of bedsores
92. The simplest manipulations are:
A. Compress, intracutaneous injection
B. Compress, jars, subcutaneous injection
C. *Mustard plasters, ice-bag, hot-water bottle.
D. Mustard plasters, jars, intramuscular injection.
E. All the previous procedures
93. Implication for mustard plasters is:
A. Postinjection infiltration.
B. Low blood pressure crisis
C. High fever.
D. *Sharp and chronic inflammatory processes of upper respiratory tract, pneumonia
E. Arthritis, myosities
94. While application of mustard plasters, the average duration of the procedure is:
A. 5-7 minutes
B. 7-10 minutes
C. *10-15 minutes
D. 15-19 minutes
E. 20-22 minutes
95. Warm compress is used for the purpose of:
A. To diminish the edema in patient with commotion of soft fabrics.
B. To diminish the pain syndrome (sharp inflammatory process).
C. *Of resolution of posinjuctive infiltrate
D. To stop the bleeding.
E. To diminish abscess intensity
96. Induction of medications through the respiratory tracts of patient belongs to :
A. Internal method of inducing
B. *External method of inducing
C. Parenteral inducing
D. Peroral inducing (through a mouth).
E. Enteral inducing
97. A cold compress is used for treatment:
A. Ischemias of soft tissue in the place of application of compress
B. Arthritises, myosities
C. *Of the initial stage of infiltrate of soft tissue
D. Neuritises
E. To the stomach-ache of uncertain character.
98. The duration of application of a warm compress is:
A. 30-40 minutes
B. 40-60 minutes
C. 1-2 hours
D. 4-6 hours
E. *8-10 hours
99. We use the ice-bag at the case of:
A. Ulcer of stomach
B. Kidney colic
C. *A prophylaxis of edema of soft tissue in the postoperative wound.
D. Acute radiculitis
E. Stomach-ache
100.
Duration of procedure of ice-bag application is:
A. *1 hour
101.
The normal temperature of the human body is :
A. 36,0-36,5 C°
B. 36,0-36,6 C°
C. 36,2-36,6 C°
D. *36,2-37,0 C°
E. 36,2-37,1 C°
102.
The nurse in an admitting department is going to measure the temperature of the
patient with fever. She cleaned the inguinal area with alcohol before manipulation. Explain
why this procedure has been done:
A. *For the purpose of providing a hygienical regime of procedure.
B. For the purpose of providing a more exact measuring of temperature.
C. For the best fixing of thermometer in ingiunal area.
D. All answers are correct
E. There is no right answer
103.
The result of temperature measurement of a patient is determined after:
A. 3-5-minute after beginning
B. 5-7-minute after beginning
C. 6-8-minute after beginning
D. *7-10-minute after beginning
E. 12-15 minute after beginning.
104.
Fever is generally a temperature above
A. *38 C°
B. 38,2 C°
C. 38,4 C°
D. 38,6 C°
E. 39 C°
105.
Infections can be controlled in
A. Hand hygiene
B. Disinfection.
C. Sterilization
D. Barriers ( e.g. gloves )
E. *All previous manipulations
106.
Diseases that are suspected to be transmitted by direct contact are
A. Clostridium dificile
B. Colonisation of infection caused by multidrug-resistant organism
C. Respiratory syncytial virus
D. *Each from all previous agents
E. Any from all previous agents
107.
Obligative intervention for the client with infection is
A. Monitor temperature as frequently ass is appropriate
B. Monitor blood pressure, pulse, respiration.
C. Monitor skin color
D. Monitor for intake and output
E. *All previous interventions
108.
Diseases that are known or suspected to be transmitted by air are
A. M. tuberculosis
B. Varicella
C. *All previous bacteria
D. Any from all previous bacteria
109.
What data are not included in hereditary history?
A. Diseases of members of the family
B. *Age of the patient
C. The ultimate cause of the deaths of the family members
D. Inherited diseases in family
E. Cases of early childhood deaths in the family
110.
What does “anamnesis morbi” mean?
A. History of life
B.
C.
D.
E.
111.
A.
B.
C.
D.
E.
112.
A.
B.
C.
D.
E.
113.
A.
B.
C.
D.
E.
114.
A.
B.
C.
D.
E.
115.
A.
B.
C.
D.
E.
116.
A.
B.
C.
D.
E.
117.
A.
B.
C.
D.
E.
118.
A.
B.
C.
*History of present illness
Family history
Past history
Chief complaints
What does “anamnesis vitae” mean?
Family history
History of present illness
*History of life
Past history
Chief complaints
What data are not included in family history?
*What infectious diseases did the child have
Age of the parents
Diseases of the members of family
Information about hereditary diseases in the family
Diseases of patient’s siblings
What data are not including in allergological history?
Allergic diseases of the members of family
What allergic diseases are the parents suffering from?
*The occupation of the patient
Has the patient ever had allergic reaction for any drug?
Has the patient ever had allergic reaction for any food (e.g. oranges, strawberry)?
What parts does anamnesis vitae consist of?
Obstetric history; family medical history; history of present illness; social history
Nutritional anamnesis; previous illnesses history; allergological history.
*Obstetric history; growth and development; previous illnesses, operations, or injuries;
allergological history; epidemiologic history; habits; social history
General/identifying information; chief complaints; present illness; past history; review
of systems; nutrition history; social history.
systems rewiew, general complaints, history of present illness
What data are not included in epidemiologic history?
Contact with children who had infectious diseases during previous 21 days.
Diarrhea during previous 7 days
What infectious diseases did the child have?
Immunizations
*The occupation of the patient
What method of examination is useful for determining the texture of the skin?
*Palpation
Auscultation
Inspection
Percussion
Interviewing
What method of examination is useful for determining the murmurs in the heart?
*Auscultation
Palpation
Inspection
Percussion
Interviewing
What method of examination is useful for determining the rales in the lungs?
*Auscultation
Palpation
Inspection
D. Percussion
E. Interviewing
119.
What method of examination is useful for determining the color of the skin?
A. Palpation
B. *Inspection
C. Auscultation
D. Percussion
E. Interviewing
120.
The most serious problem that wrinkles in the bedclothes is:
A. restkessness
B. sleeplessness
C. *decubitus ulcer
D. bleeding
E. shock
121.
Scabies is caused by:
A. Pediculus humanus capitis
B. *Sarcoptes scabiei subspecies hominis
C. Candida albicans
D. Candida lusitaniae
E. Streptococcus pyogenes
122.
What are bedsores (pressure ulcers)?
A. *All answers are right
123.
Prescribe substance to eradicate lice and nits
A. *0.15 % carbophos-water emulsion
B. shampoo “Head@shoulders”
C. “Soframycin”
D. “Levomycol”
E. All answers are right
124.
In admissions office during examination of patient’s scalp the nurse found whitish oval
specks adhering to hair shaft. Prescribe treatment
A. antidandruff shampoo
B. 0.15 % carbophos water-emulsion
C. “Soframycin”
D. “Levomycol”
E. *shampoo “NITTIFOR” for 20 min and rinse with 6 % solution of table vinegar
125.
Prescribe substance to eradicate lice and nits.
A. *shampoo “NITTIFOR”
B. antidandruff shampoo
C. “Soframycin”
D. “Levomycol”
E. All answers are right
126.
What recommendation is not used for prevention of pressure ulcers in a patient in
coma?
A. Change child’s position not less than 1 time per 3 hours
B. *Help with surgical “cleaning” of affected place, then cover the wound with sterile
dressing soaked with 1 % solution of potassium permanganates; change the dressing 2-3
times per day
C. Keep clothing and linen clean and dry
D. Maintain meticulous skin clearness
E. Ultraviolet radiation (UVR) on the problem parts of skin
127.
During inspection of scalp for infestation by Pediculus humanis capitis observe for:
A. empty nit cases
B.
C.
D.
E.
whitish oval specks adhering to hair shaft
erythema and excoriations
clusters of small erythematic papules on the scalp skin
*any movement that can indicate a louse, whitish oval specks adhering to hair shaft, and
empty nit cases
128.
How many specimens should be obtained for adhesive tape test for enterobiosis?
A. Three specimens per day
B. One specimen
C. *Three consecutive specimens
D. Three specimens, one per week
E. Four consecutive specimens
129.
What is the preferred diagnostic screening test for ascaridosis?
A. *Throat culture
B. Stool culture
130.
When should adhesive tape test specimens be obtained?
A. After defecation
B. Before going to sleep
C. After breakfast
D. When the patient first awakens in the morning, after washing
E. *When the patient first awakens in the morning, before washing.
131.
To measure the nasogastric tube for the exact distance it is necessary to measure the
distance:
A. *from midway between the belly button and the highest point of the lower rib cage to
the child’s ear lobe then out to his nose (his head turned to the side)
B. from the belly button to the child’s ear lobe.
C. from the belly button to the child’s nose (his head turned to the side).
D. from midway between the belly button and the highest point of the lower rib cage to the
child’s nose (his head turned to the side)
E. from midway between the belly button and the highest point of the lower rib cage to the
child’s lower incisors
132.
What is the correct place for the compressions during CPR?
A. upper 1/3 of sternum
B. *middle 1/3 of sternum
C. epigastric area
D. lower 1/3 of sternum
E. umbilical area
133.
The basic patient’s rights are:
A. Right to be treated
B. Right to refuse treatment
C. Right to be informed
D. *All of them
E. Right to be treated and informed
134.
The MOST serious problem that wrinkles in the bedclothes can cause is
A. restlessness
B. sleeplessness
C. *decubitus ulcers
D. bleeding
E. shock
135.
An important way to reduce the incidence of decubitus ulcers is to:
A. keep the client in bed
B. force fluids every 2 hours
C. *change position every 2 hours
136.
137.
138.
139.
140.
141.
142.
143.
144.
D. all of the above
E. none of the above
In caring for a confused elderly man, it is important to:
A. *keep the bedrails up except when you are at the bedside
B. close the door to the room so that he does not disturb other patients
C. keep the room dark and quiet at all times to keep the patient from becoming upset
D. remind him each morning to shower
E. all of the above
Before assisting a client into a wheelchair, the FIRST action would be to check if the
A. client is adequately covered
B. floor is slippery
C. door to the room is closed
D. *wheels of the chair are locked
E. none of the above
The hot water bottle is an example of
A. *local dry heat application
B. generalized dry heat application
C. local moist heat application
D. generalized moist heat application
E. none of the above
Clients receiving an enema are usually placed
A. *on the left side
B. flat on the back
C. in a semi sitting position
D. none of the above
E. supine position
The most common site for counting the pulse is
A. carotid artery
B. femoral artery
C. brachial artery
D. *radial artery
E. none of the above
Bedsores may develop in clients who are
A. incontinent
B. paralyzed
C. poorly nourished
D. *all of them
E. none of the above
The FIRST step in performing any procedure is to:
A. *wash your hands
B. gather needed equipment
C. explain the procedure
D. provide privacy
E. none of the above
A decubitus ulcer is caused by
A. poor circulation
B. pressure on the skin
C. poor nutrition
D. *all of the above
E. none of the above
The MAIN safety factor in transferring a client is to
A. always transfer slowly
B.
C.
D.
E.
*raise the side rails
secure adequate assistance
make sure the client is not tired
none of them
145.
The MOST accurate method of measuring body temperature is
A. *oral
B. axial
C. feeling the forehead
D. none of them
146.
Microorganisms can be spread by direct and indirect contact. An example of indirect
contact is
A. bathing the patient
B. using contaminated blood
C. *touching objects or dirty instruments
D. breathing dust particles in the air
E. none of them
147.
Which of the following helps prevent decubitus ulcers?
A. bed rest
B. liquid diet
C. *back rubs
D. support hose
E. none of them
148.
A cane, walker, and crutches are all used to help a client walk. What else do they have
in common?
A. Clients must know how to use them correctly
B. They all must have rubber tips to prevent slipping
C. They all must be fitted to the client
D. *all of the above
E. none of the above
149.
An example of contamination through indirect contact is
A. bathing the resident
B. *touching instruments or linens the resident has used
C. the resident sneezing or coughing on you
D. all of the above
E. none of the above
150.
Another name for urination is
A. defecation
B. *voiding
C. wetting the bed
D. flatus
E. none of them